Questions from Mock Exams Flashcards

You may prefer our related Brainscape-certified flashcards:
1
Q

CONCEPT: Standard Deviation
Your client, Alice, owns the following four different diversified mutual funds:

Growth fund - $45,000
Emerging market fund - $14,000
Government bond fund - $50,000
Corporate bond fund - $35,000

Alice is concerned about overall portfolio risk. She is concerned about standard deviation and other factors. Due to a recent inheritance, she has additional money to invest. To which among her currently held mutual funds do you suggest she add money?

A

The emerging market fund currently represents is the smallest percentage of the portfolio allocation and likely has the lowest correlation coefficient relative to the other funds. Reducing correlation coefficient would reduce the portfolio’s overall risk. The correlation coefficient to be highly important for the exam. (Somewhat subjective.)

How well did you know this?
1
Not at all
2
3
4
5
Perfectly
2
Q

QUESTION: When, if ever, can a corporation that issues qualified stock options (ISOs) receive a tax deduction for the ISOs?

A. Never
B. Always
C. Yes, if the ISO is disqualified
D. Yes, if the ISO is qualified
E. Yes, if no more than $100,000 worth of ISO stock is granted that vests in a specific year

A

ANSWER: C. If the stock that was acquired under the option (right to buy) is sold before the two year /one year holding period, the excess of the fair market value of the shares at the time of exercise over the exercise price is treated as compensation to the option holder. That creates a corresponding deduction for the issuing corporation.

How well did you know this?
1
Not at all
2
3
4
5
Perfectly
3
Q

QUESTION: Todd wants to defer the distributions from the money purchase plan in which he participates for as long as possible. He works for RJ, Inc. RJ wants him to continue working for it beyond the plan’s stated retirement age 65. If he continues to work beyond 72 and contribute to the plan, what is the latest time when he can take his first distribution and not be penalized?

A. When he attains age 72
B. By April 1st of the year after he turns 72
C. When he retires from his job with RJ, Inc.
D. By April 1st of the year following the year when he retires from his job with RJ, Inc.

A

ANSWER: D. Todd is a rank-and file-participant in the money purchase plan and clearly not a 5% owner. Thus, he may delay his required beginning date (RBD) from the plan until April 1 of the year following the year when he retires from service with this employer.

How well did you know this?
1
Not at all
2
3
4
5
Perfectly
4
Q

QUESTION: When you met with John and Jodi Adams for your regular monitoring meeting, they provided you with information about new developments in their lives. After you congratulate them they ask you to help them prioritize the reasons for making changes to the original financial plan that you wrote for them. How would you rank the changes listed below in order of importance from highest to lowest?

I. They inherited money from Jody’s mother
II. Jody is expecting a second child in 2 months
III. John just received new job promotion which entails a move to an adjacent state (50 miles away).
IV. The adjacent state has a high state income tax

A. I, II, III, IV
B. II, III, IV, I
C. III, I, IV, II
D. IV, II, I, III

A

ANSWER: A. Identify the most important and the least important reasons to modify the original plan. The Adams’s will need a plan for the inherited money. The state level income tax differential is likely to be small. If the Adams’s itemize, it may produce an itemized deduction. Because the new baby is a second child, they have already considered the financial planning that accompanies parenting. (In ranking questions, identifying the “most” and “least” generally leads you to the answer: The middle choices are often too similar to differentiate.

How well did you know this?
1
Not at all
2
3
4
5
Perfectly
5
Q

QUESTION: You are a CFP® certificant and Todd and Belinda Harding are your new clients. During the initial interview Todd excuses himself for a restroom break. Belinda whispers to you that Todd is a compulsive gambler. She confides in you that she has managed to squirrel away a significant amount of cash that at the moment is in a money market account. Belinda asks you not to tell Todd about the account and says that she wants to call you the following morning to discuss allocation options for the money. How should you best handle this awkward situation?

A. When Todd returns to the room and as you begin to gather data step, act as if you have no awareness of the account that Belinda mentioned.
B. Speak with Belinda the following morning to discuss asset allocation choices.
C. Terminate the relationship before you proceed to the data gathering step in the financial planning process.
D. When Todd returns from the bathroom tell him about the account.

A

ANSWER: The correct answer is C. Who is the client? The presumption was that both Todd and Belinda would become your clients (not just Belinda). This is the initial interview (going to be clients). Regarding Answer D Belinda makes clear that she does not want her compulsive gambler husband to know about the account. Further, without the disclosure of the value of assets –including the money market account, the data is too vague for meaningful financial planning.

How well did you know this?
1
Not at all
2
3
4
5
Perfectly
6
Q

QUESTION: Your client, Dennis Hart explains to you that he wants a reasonable level of income but also some long-term growth. If you believe that he can address both of his investment objectives, which of the following securities would you suggest to Dennis?

A. Convertible bonds
B. Preferred stocks
C. Blue chip stocks
D. Corporate commercial paper

A

ANSWER: The best answer is A. Most logical investors will accept a lower interest rate in exchange for the potential price appreciation from converting the bond if the prices of the issuers’ stocks rise above the bond’s conversion price. Preferred stock is regarded as a fixed income investment with little growth potential. Many blue-chip stocks distribute small dividends and they can be skipped in a profit-less year.

How well did you know this?
1
Not at all
2
3
4
5
Perfectly
7
Q

QUESTION: Smokestack Inc. voluntarily terminated its defined benefit plan. Your client, Homer Connors, age 61, has been a long-time employee of Smokestack, Inc. and a participant in this pension. The “termination” has made Homer quite anxious. What might you tell Homer that may make him feel less anxious?

A. The 10% penalty (59½ year rule) will not apply to distributions.
B. The account balance must be rolled over into an IRA account.
C. Homer is 100% vested.
D. The plan is fully funded. There is no need to worry.

A

ANSWER: C. The 10% penalty will not be imposed on Homer because he is over age 59½ and is a possible answer. The plan is fully funded at normal retirement age, not necessarily at a premature termination. Homer would get the account balance that is attributable to him and be fully vested.

How well did you know this?
1
Not at all
2
3
4
5
Perfectly
8
Q

QUESTION: Mrs. Smith, age 80, is comparing different investment portfolios. The thought of losing principal makes her very uncomfortable. While she would appreciate some income from her investments, that is a secondary concern. After listening to her carefully, which of the following portfolios would you suggest?

A. 10% money market mutual funds, 10% blue chip common stocks, 80% long-term bonds
B. 50% bank issued CDs, 50% long-term investment grade corporate bonds
C. 10% money market mutual funds, 10% blue chip common stocks, 80% investment grade short-term bonds
D. 10% money market mutual funds, 40% bank issued CDs, 50% investment grade long-term bonds

A

ANSWER: The best answer is C. Due to their high durations, the long-term bonds carry significant principal risk if interest rates rise. The short-term bonds (80%) along with only 10% in quality common stock seems reasonable given her fear of principal loss and desire for income.

How well did you know this?
1
Not at all
2
3
4
5
Perfectly
9
Q

QUESTION: Your client, Jane Thompson is divorced. Her ex-husband Alex Thompson is now remarried to Lola, age 25. Lola is an exotic dancer. Since he married Lola, Alex has been a bit tardy on making alimony payments. Jane wants you, her financial planner, to meet with Alex. Jane is willing to pay for your services and Alex is willing to meet with you. What should you do?

A. Tell Jane that you cannot meet with Alex because there is a conflict of interest.
B. If you do see Alex, do not discuss Jane’s financial affairs with him.
C. Tell Jane that Lola needs to be included in the conversation.
D. Tell Jane that the best solution is to refer Alex to another financial planner.

A

ANSWER: The best answer is D. Jane and Alex want help but you don’t want to be in an awkward situation. Answer D does provide help regarding the situation. Lola is not a party to the alimony agreement between Jane and Alex.

How well did you know this?
1
Not at all
2
3
4
5
Perfectly
10
Q

QUESTION: You, a CFP certificant, are having a first meeting with Will, an energetic, young client. He is 28 and a promising entrepreneur. He says, “I do not want a whole song and dance, I just want to invest some money. Can you help me with that?” What should you do next?

A. Create an asset allocation model for Will.
B. Because he has a business, recommend that he open an IRA.
C. Check that his insurance coverages are adequate.
D. Calculate and analyze his cash flow.

A

ANSWER: D. While Answer C may be arguable, D is a better choice. If Will turns out to have negative cash flow, he should be addressing that before he invests. Answer A depends on Answer D. The insurance answer is too vague, especially without knowing more about Will’s family situation.

How well did you know this?
1
Not at all
2
3
4
5
Perfectly
11
Q

QUESTION: Sally donates several bags of old clothes to the Salvation Army. Which statement below best reflects the documentation that Sally would need in order to claim a charitable income tax deduction?

A. Deduction of up to $250 does not require a receipt.
B. Deduction of $250 but less than $1,000 must be documented.
C. The deduction is the lesser of fair market value or the donor’s basis (substantiated).
D. The deduction is limited to basis (unsubstantiated).

A

ANSWER: The best answer is C. For charitable gifts of less than $250, a dated receipt is proof for purposes of an income tax deduction. The receipt should c include a description of the property. A written receipt would list the items donated with a corresponding value. Sally should keep records showing the fair market value and her cost basis. For charitable gifts exceeding $250 Sally must substantiate the deduction by written acknowledgement from the charity. Cash donations up to $300 single/$600 joint do not have to be documented for 2021 if you take the standard deduction.

How well did you know this?
1
Not at all
2
3
4
5
Perfectly
12
Q

A CFP® professional meets a prospective client who is prepared to discuss his retirement accounts that were provided through a former employer. The client is concerned that the investments in those accounts are too aggressive. He states that he and his wife would like to retire when they are 65 and that they have recently been writing many checks from their account to support their adult son who can’t seem to find a job. After further analysis, the CFP® professional determines that the client does not have enough cash flow to retire when they turn 65 while continuing to support their son. How should the CFP® professional proceed?

A. Communicate to the clients that their retirement goals are unrealistic: that they should plan to work until age 70, and that they should stop giving money to their son.
B. Help the clients to review and prioritize goals.
C. Review the client’s current and potential income streams to identify ways to solve the immediate cash flow shortfall.
D. Recommend a conservative asset allocation model to reduce the risk in his retirement accounts and suggest that they stop giving money to their son.

A

ANSWER: B. The CFP Board would feel that it is important to encourage the client to reassess priorities before making specific recommendations. Answer C is wrong because it assumes the client has established new goals. Answers A and D are arguable.

How well did you know this?
1
Not at all
2
3
4
5
Perfectly
13
Q

Your married clients, Adam and Jane Smith, have provided you with the following tax information for the current year.
Adam’s salary (net of his 401(k) deferral) - $135,000
Jane’s income (from babysitting) - 1,000
Real estate income (active participation) - 5,000
Dividends - 1,000
Adam’s IRA contribution - 6,000
Jane’s IRA contribution - 6,000

What is the amount of their current year AGI?

A. $124,000
B. $130,000
C. $136,000
D. $142,000

A

ANSWER: C.
Adam $135,000
Jane 1,000
Real estate 5,000
(income) Dividend income 1,000
IRA - 6,000
AGI - $136,000
Adam has a 401(k) and is above phase out ($109K - $129K) for IRA deductibility. However, Jane can contribute and deduct the $6,000. The spousal IRA phase out is at $204,000.

How well did you know this?
1
Not at all
2
3
4
5
Perfectly
14
Q

QUESTION: Mrs. Spellman has come to you for advice. Her current net worth is about $450,000. She says she could use “more spending money.” Which of the following techniques would increase Mrs. Spellman’s cash flow?

A. Take an equity loan against her home ($150,000 FMV with a basis of $30,000)
B. Sell her vacation home and invest the net proceeds in municipal bonds
C. Sell investment property that is producing minimum net income ($8,000/yr.). ($200,000 FMV, original cost $100,000 but fully depreciated)
D. Sell non-income producing land ($100,000 FMV, basis $150,000)

A

ANSWER: B. In Answer A, the loans produce negative cash flow because they carry interest. The sales of the home and the investment property do not increase cash flow. We do not know how the money will be invested. The question focuses on increasing cash flow.

How well did you know this?
1
Not at all
2
3
4
5
Perfectly
15
Q

If a taxpayer is subject to AMT, which of the following could reduce the AMT payable?
I. Exercise nonqualified stock options
II. Take short-term capital gains
III. Delay until next year the payment of a property tax bill
IV. Exercise and sell an ISO in the year of exercise

Answers:
A. I, II, III, IV
B. II, III, IV
C. III, IV
D. II and III
E. I and IV

A

ANSWER: Increasing taxable income (Answers I and II) for regular tax purposes until it reduces or eliminates AMT exposure. Delaying certain itemized deductions such as medical expenses, charitable gifts and local property tax creates more regular income. An ISO exercise adds to AMT income, but that addition is nullified by a disqualifying disposition such as a sale in the year of exercise).

How well did you know this?
1
Not at all
2
3
4
5
Perfectly
16
Q

QUESTION: Sidney is very displeased with a particular CFP® practitioner’s recommendations. He strongly believes that they were unsuitable and resulted in unnecessary losses. Further, Sidney later learned that the planner did not provide him with adequate disclosure of conflicts of interest and other matters. To which the following parties should Sidney send his letter of complaint?

Answers:
A. The CFP® Board
B. The planner’s supervisor
C. FINRA
D. The SEC

A

ANSWER: A. We know that the planner is a CFP®. Nothing in the question indicates FINRA registration. Nor does the question indicate that the planner is a federal covered advisor regulated by the SEC. The planner may or may not have a direct supervisor. The culture of the exam is that CFP Board wants to know of the complaints, then they can investigate them.

How well did you know this?
1
Not at all
2
3
4
5
Perfectly
17
Q

QUESTION: An employer can self-fund certain benefits under a 501(c)(9) voluntary employees’ beneficiary association (VEBA). Which of the following may be funded?

I. Death benefits
II. Medical benefits
III. Unemployment benefits
IV. Retirement benefits
V. Deferred compensation benefits

A. I, II, III, IV
B. I, II, III
C. I, II
D. IV, V
E. All of the above

A

ANSWER: B. Retirement and deferred compensation benefits may not be funded through a VEBA.

How well did you know this?
1
Not at all
2
3
4
5
Perfectly
18
Q

QUESTION: George Hallas owns 80% and his daughter, Georgina 20% of Hallas, Inc. (a corporation). Hallas, Inc. grosses approximately $20 million in a typical year. George and his daughter also own a general partnership worth $5 million. George owns a $3 million life insurance policy outright under which he is the named insured. He wants to remove the life insurance policy from his estate. What do you recommend?

A. Sell the policy to the corporation for buy-sell purposes.
B. Sell the policy to the partnership for buy-sell purposes.
C. Transfer the policy to the partnership for buy-sell purposes.
D. Gift the policy to his daughter.

A

ANSWER: D. If the corporation owns the policy, the proceeds may be considered in valuing the decedent’s interest for federal estate tax purposes unless there is valid agreement fixing the price that would reflect an arms-length sale to an unrelated party (questionable because the buyer and seller are daughter and father, respectively. Answers B and C create a similar problem. When George dies the partnership dissolves. The ownership of the policy after that point would be uncertain and possibly flow through to George’s estate.

How well did you know this?
1
Not at all
2
3
4
5
Perfectly
19
Q

QUESTION: Joe Jones works for “Take-A-Boat” boat rentals. The company has a SIMPLE plan in which Joe participates. As he approaches 72, plans on working fewer hours. He would still like to participate in the SIMPLE plan. Which of the following can you accurately tell Joe?

A. You cannot contribute to a SIMPLE IRA after age72; but you can contribute to a non-deductible IRA
B. You can continue to contribute to a SIMPLE.
C. Since you are a more than 5% owner you must start to take distributions from the SIMPLE when you reach 72 and then stop contributing.
D. You can contribute to a Roth IRA.

A

ANSWER: B. We do not know Joe’s AGI. AGI affects Answer D. If Joe’s AGI is over the threshold, he may not contribute to a Roth. As an employee, Joe must take distributions from the SIMPLE when he reaches 72, however, he can still contribute. (Yes, money is flowing out and into Joe’s account in the same year(s).

How well did you know this?
1
Not at all
2
3
4
5
Perfectly
20
Q

QUESTION: Baker, Inc. provides a qualified retirement plan (employer funded). The plan falls under numerous ERISA rules. The plan lost 50% due to poor investment decisions in the previous year. What recourse can the employees take?

A. Sue the plan officials for 100% of the investment losses.
B. Sue the plan officials for 50% of the 50% loss.
C. Do nothing: qualified plan investment managers are not required to make profits.
D. Sue the plan officials for 100% of the losses plus punitive damages
E. Sue the plan officials for losses to the plan

A

ANSWER: E. Errant plan officials can be held personally liable for losses to the plan as well as other factors. ERISA prohibits monetary punitive damages for claims.

How well did you know this?
1
Not at all
2
3
4
5
Perfectly
21
Q

QUESTION: Pension contributions are based on compensation. Which of the following is generally considered to be compensation to an employee?

I. Salary
II. Bonus
III. Business expense reimbursement
IV. Incentive stock options
V. Contributions to a deferred compensation plan

A. All of the above
B. I, II, IV
C. I, II, III
D. I, II
E. I, IV

A

ANSWER: D. Salaries and bonuses are clearly compensation. A reimbursement pays the employee back for business expenses incurred but is not compensation. An ISO is a right to buy the employer’s stock and is not compensation. However, if the ISO becomes disqualified because the stock is exercised and sold in the same calendar year, the employee may be required to recognize any profits as compensation. Deferred compensation is not treated as compensation until it is constructively received. For tax purposes, compensation is considered current when it is paid no later than 2½ months after the year in which it is earned. For a more-than-50% owner, the compensation must be paid by year end. Deferred compensation is not compensation for tax purposes until it is constructively received.

How well did you know this?
1
Not at all
2
3
4
5
Perfectly
22
Q

QUESTION: Duggan is a 70-year-old man about to retire. If he annuitizes the current account balance from the retirement plan in which he has participated throughout his long career, which of the following annuity options will provide Pat with the highest payment in his first year of retirement?

A. Life income with a 5-year period certain
B. A life annuity
C. Joint lifetime income with his son (age 49)
D. Joint life income with his wife (age 71)

A

ANSWER: B. The life annuity always produces the highest payout. A life annuity may also be called as a pure life annuity or a straight life annuity.

How well did you know this?
1
Not at all
2
3
4
5
Perfectly
23
Q

QUESTION: Which of the following statements is true a regarding a QPRT if the grantor dies during the retained-interest term?

A. The value of the remaining term will return to the grantor’s gross estate.
B. It leaves the grantor’s estate with no greater tax liability than had the QPRT had not been established.
C. The applicable credit amount plus any gift tax actually paid on the original transfer are lost.
D. The present value of the retained income interest is brought back into the gross estate.

A

ANSWER: B. The full value of the home generally reflecting date of death FMV is brought back into the gross estate. Let’s say you transferred a home worth $1 million under a 10-year QPRT. Had the QPRT never been established the estate would have the same tax exposure because the property would appear in the gross estate at FMV.

How well did you know this?
1
Not at all
2
3
4
5
Perfectly
24
Q

QUESTION: A premature distribution penalty tax applies to which one of the following IRA distributions?

A. A distribution made to the owner ($10,000 lifetime limit) for the primary residence
B. A distribution made to the owner for qualified higher education expenses furnished to the owner personally
C. A distribution made to a 50-year-old beneficiary after the death of the owner
D. A distribution attributable to the owner’s disability

A

ANSWER: A. Don’t get tricked! The distribution must be for the purchase of a first home, not necessarily a primary residence.

How well did you know this?
1
Not at all
2
3
4
5
Perfectly
25
Q

QUESTION: Arthur, age 63 regrets retiring early. He’s single and bored. Arthur found a job at Walmart as a greeter. The job will pay $15,000 per year. Arthur doesn’t need the money because he is currently receiving $6,000 per month from his former employer’s money purchase pension plan plus early Social Security retirement benefits of $1,000 per month. Arthur lives in a comfortable apartment, has full medical coverage and makes no charitable contributions. He normally claims the standard deduction. Which of the following is true if he takes the job with Walmart?

A. He will remain in a 22-24% income tax bracket.
B. He should find a job that pays him more than the minimum wage ($15.00/hour).
C. The impact of the earned income will be a very marginal increase in income tax.
D. His Social Security Retirement benefits could be reduced because of his earned income.

A

ANSWER: A, Arthur will be in the 22-24% bracket. If he works the same hours for better pay, Arthur will exceed the 1-2 rule ($19,560) which would reduce his current Social Security retirement benefits. For now, Arthur remains with Walmart.

How well did you know this?
1
Not at all
2
3
4
5
Perfectly
26
Q

QUESTION: Plant Parenthood is a landscaping company, it has 18 full-time employees participating in is group health plan, and 4 full-time employees who are not participating in the plan. Joe, a participating employee with family medical coverage under Plant Parenthood’s group health insurance plan, just divorced Sara. How long will COBRA cover Sara and Debbie (Joe’s 12-year-old daughter)?

I. Sara is entitled to 18 months of continuation in the group plan.
II. Sara is entitled to 36 months of continuation in the group plan.
III. Debbie is entitled to 18 months of continuation in the group plan.
IV. Debbie is entitled to 36 months of continuation in the group plan.
V. Debbie is still covered under the plan group medical insurance plan.

A. I, III
B. II, IV
C. II, III
D. I, V
E. II, V

A

ANSWER: E. Under COBRA rules, Sara will get 36 months of continuation coverage in the group medical insurance plan (due to divorce). Debbie is still Joe’s daughter and will continue to be covered by his group health plan (family plan). The plan is subject to COBRA requirements because Plant Parenthood has 22 full-time employees

How well did you know this?
1
Not at all
2
3
4
5
Perfectly
27
Q

QUESTION: Terrie Cross and Brenda Davis have decided to close their business. They had entered into a cross-purchase buy sell agreement funded with life insurance policies. Both Terrie and Brenda are married. How should they handle the ownership of their policies at this point?

A. Each should purchase her own policy from the other owner.
B. They should maintain the current ownership arrangements of the insurance policies.
C. Terrie should sell Brenda’s policy to Brenda’s husband, and Brenda should sell Terrie’s policy to Terrie’s husband.
D. They should change to an entity buy sell agreement.

A

ANSWER: A. After the business closes, it is reasonable that each owner then owns the policy on her own life. Because the business is closed, there is no reason to maintain the current arrangement of ownership of the policies. Answers C and D trigger “transfer for value”.

How well did you know this?
1
Not at all
2
3
4
5
Perfectly
28
Q

QUESTION: Mr. Smith is subject to the AMT. Which of the following can reduce his AMT payable?

A. Exercising more nonqualified stock options
B. Assuming a larger mortgage
C. Purchasing more municipal bonds (private activity)
D. Buying an oil and gas partnership
E. Purchasing more public purpose bonds

A

ANSWER: A. Exercising nonqualified stock options will increase his regular income which thus reduces his AMT payable. The mortgage interest deduction associated with a larger mortgage will decrease his taxable income. Purchasing public purpose bonds will have no effect.

How well did you know this?
1
Not at all
2
3
4
5
Perfectly
29
Q

QUESTION: Toby Smith, age 61, gifts $1 million to an irrevocable trust that provides income only to his troublesome son, Bugsy, age 37. Bugsy can’t keep a job and is always asking his father and others. The trust income is distributed quarterly. Toby’s investment advisor handles the $1 million trust portfolio. The payout is approximately 3% or $30,000. Toby is married with three other children. The other children are upset because no trust arrangement established for them. Which of the following statements accurately reflects Toby’s situation?

A. Toby has made a taxable gift of $1 million.
B. Toby and his wife have made taxable gifts of $484,000 (split-gift less $16,000)
C. The $30,000 is taxable income to Toby.
D. Toby should have established a 2503(c) trust.

A

ANSWER: A. Gifts in trust are future interest gifts. No Crummey powers are included. The income is taxable to Bugsy. This is a 2503(b) trust. The trust is not tainted; nothing indicates that Toby is the trustee. Toby transferred the money into the trust. Nothing indicates a split gift. There is no Crummey provision.

How well did you know this?
1
Not at all
2
3
4
5
Perfectly
30
Q

QUESTION: Which of the following types of qualified retirement plans can be integrated with Social Security?

I. A 401(k) plan (no match or company contribution)
II. A money-purchase plan
III. An ESOP
IV. A stock bonus plan
V. A defined benefit plan

A. I, II, III, IV
B. II, IV, V
C. III, V
D. II, V
E. All of the above

A

ANSWER: B. ESOPs and 401(k) plans with no employer matching contributions cannot be integrated with Social Security. Item I does indicate a pure 401(k) with no match or profit sharing contribution plan. Defined benefit and money purchase pension plans can be integrated. Stock bonus plans can be integrated.

How well did you know this?
1
Not at all
2
3
4
5
Perfectly
31
Q

QUESTION: Bill Williams, CFP®, wrote a plan for his client, Sally Linton, age 58. Sally indicated she would work for 9 more years until her FRA (full retirement age) to qualify for full Social Security and maximum qualified plan benefits. One month later Sally unexpectedly quit her job. The facts are, however, that the company she worked for was sold to a competitor and her position was eliminated. The new company offered her an unacceptable position. Now Sally realizes, at 58, with outdated skills, retirement is her only option. Sally indicates to Bill she is willing to sell her second home at the beach. How should Bill proceed?

A. Make no recommendations until Sally sells the second home.
B. Advise Sally to go back to her employer and take the position.
C. Review Sally’s current lifestyle and expenses and establish a budget until the second home sells.
D. Advise Sally to apply for unemployment benefits.
E. Ask Sally to come back in a few days. The planning needs to be reevaluated.

A

ANSWER: C. Bill has all of Sally’s data. He just completed a plan. Sally is upset, she needs advice now. Making her wait might make her more upset. In creating the plan, Bill did assess Sally’s current lifestyle and expenses. Sally can’t file for unemployment insurance benefits because she terminated voluntarily. Sally clearly does not want to take the newly offered position.

How well did you know this?
1
Not at all
2
3
4
5
Perfectly
32
Q

QUESTION: Your client, Mrs. Cates, died 6 months ago. Her family inherited almost $5 million without shrinkage from federal estate tax. Mrs. Cates property generally received a step-up in basis. Her son, Caleb, received $2 million from his mother’s estate. Caleb deposited the money into a joint account that he and his wife have maintained for years. Caleb and his wife, Cindy, had been your clients before Mrs. Cates died. The account, which holds other assets in addition to the inheritance has a current FMV of $3.5 million. This morning Caleb called to request $100,000 in cash, (not a check) from the account. This is a very unusual request, so you ask Caleb the reason for the withdrawal. He says he needs it to pay his mistress in exchange for her agreeing not to tell Cindy about the affair. What should you do?

A. Call Cindy for authorization to make the distribution (joint account)
B. Tell Caleb that you will ignore federal money laundering rules, and hide the distribution from your compliance officer.
C. Tell Caleb that this situation creates a conflict of interest for you and that and cannot proceed with the withdrawal without Cindy’s consent
D. Terminate the relationship. Make arrangements to pay him $100,000 in the form of a joint check

A

C. This situation creates a serious conflict of interest for you because you were hired by both Caleb and Cindy. The account is joint property Due to federal money laundering rules, brokerage firms generally will not facilitate substantial distributions in cash. Obviously, Caleb does not want Cindy to learn about his request for the money.

How well did you know this?
1
Not at all
2
3
4
5
Perfectly
33
Q

Which of the following risks is not present in an investment in zero coupon bonds?

A. Interest rate risk
B. Market risk
C. Reinvestment rate risk
D. Purchasing power/inflation risk
E. Default risk

A

ANSWER: C. One advantage of a zero-coupon is the elimination of reinvestment rate risk because there is no coupon to be reinvested. The zero-coupon bond is generally subject to market risk, interest rate risk, and, if the zero is not a Treasury security,

How well did you know this?
1
Not at all
2
3
4
5
Perfectly
34
Q

Mrs. Kalish, age 82, gifted the following assets over the past three years. Three years ago, she gifted a stock portfolio with a basis of $1million and a FMV of $1.5 million currently worth $2 million. Two years ago, she placed $2 million in a 5 year GRAT with a gift tax value of $1.25 million, currently worth $2.4 million. One year ago, she gifted a whole life insurance policy with a face value of $1 million and an interpolated terminal reserve plus unearned premium of $100,000. This year, because she is sad over the passing of her pet cat, Melina, she gave $100,000 to the Society for the Prevention of Cruelty to Animals. Mrs. Kalish passed away today. Which of the following is true?

A. All the assets shown above will be included in Mrs. Kalish’s gross estate at FMV (throwback rule).
B. If her daughter sells the stock, she will have to pay tax on $500,000 at LTCG rates.
C. $3.4 million of the assets will be included in Mrs. Kalish’s gross estate.
D. The stock is not included in her gross estate because she lived 3 years following the transfer.

A

ANSWER: C. No 3-year throwback rule applies to stock. Because it was a taxable gift it will be added to the taxable estate rather than be included in the gross estate. The stock was gifted to the daughter. Thus, the daughter’s carryover basis is $1 million, which, given the $2 million in sales proceeds produces a capital gain of $1 million. The GRAT (5 year) and the life insurance (3-year rule) are included in the gross estate. The GRAT assets would be included in Mrs. Kalish’s gross estate likely at date of death FMV because she died before the end of the term of the trust. The life insurance policy would be included in her gross estate at face value because Mrs. Kalish had held incidents of ownership in the three-year period prior to her death.

How well did you know this?
1
Not at all
2
3
4
5
Perfectly
35
Q

Robert Zimmerman owns Smokey Bacon, Inc. Smokey Bacon provides a profit sharing 401(k). Robert makes the maximum elective deferral, and with the company match and typical forfeitures, annual additions have ranged between $20,000 - $25,000. He has started another company, Eggcellent Eggs, Inc. with some good friends, and they are considering a profit sharing 401(k) plan for Eggcellent Eggs. Robert will be a controlling shareholder in Eggcellent Eggs. Given Robert’s positions, which of the following statements is true?

A. Robert cannot be a participant in the Egcellent Egg’s profit sharing 401(k) plan.
B. Since Robert is fully participating in the Smokey Bacon profit sharing plan, he cannot participate in Eggcellent Eggs profit sharing plan (related employers).
C. Robert is limited to the littlest of 25% of covered compensation or $61,000 (2022) for annual additions provided by both Bacon and Eggs.
D. He cannot defer any compensation into Eggcellent Egg’s 401(k) plan.

A

ANSWER: D. He can be a participant in Eggcellent Egg’s plan for profit sharing contributions but not elect any more deferrals. Answer C is incorrect because his annual additions limit (for both plans combined) is the lesser of 100% of compensation or $61,000 in 2022 (Bacon and Eggs are related employers).

How well did you know this?
1
Not at all
2
3
4
5
Perfectly
36
Q

Your client, Byron Sheridan, died recently at the age of 83. He is married to Virginia, age 83. Byron enjoyed managing his investments and diversified his invested assets among several asset classes. When Byron died, he held the asset listed below. Which of them would be eligible for a step up in basis?

A. A $100,000 CD now worth $105,000 (acquired 5 years ago).
B. A municipal bond purchased at a discount ($95,000) two years ago now having a date of death FMV of $100,000
C. Stock purchased 6 months ago for $50,000 that created a $10,000 STCL.
D. An annuity purchased 10 years ago for $100,000 having a date of death FMV of $115,000.

A

ANSWER: B. The municipal bond was held for the long- term and is eligible for stepped up basis upon Byron’s death. The tax deferred accounts such as an annuity (and retirement accounts, generally do not step up in basis.) The CD is cash. There is no stepped up basis; a dollar is a dollar.

How well did you know this?
1
Not at all
2
3
4
5
Perfectly
37
Q

QUESTION: PDQ common stock has experienced the following returns over the past 4 years.

Year 1: -20%
Year 2: +10%
Year 3: +10%
Year 4: -40%

Which of the following is true about the standard deviation for PDQ?

A. The standard deviation is negative.
B. The information provided is insufficient to calculate standard deviation.
C. Only three years of returns can be used to determine standard deviation accurately.
D. The standard deviation of PDQ is 24.5%.

A

ANSWER: D. Calculating was not needed to answer this question. There is no limit as to the number of years of return outcomes used to determine the standard deviation of a stock. The information necessary to calculate the standard deviation for PDQ, namely return outcomes, is shown in the question data. Answer A is incorrect because standard deviation cannot be negative.

How well did you know this?
1
Not at all
2
3
4
5
Perfectly
38
Q

CONCEPT: Retirement Plan Rules
QUESTION: Brad participates in his employer’s SIMPLE plan. He is not an owner of the company. He plans to continue working indefinitely and continue making contributions to the SIMPLE. His birthday is February 16. What is his latest permissible required beginning date for distributions?

A. August 16 of the year in which he turns 72
B. April 1st of the year following year in which he turns age 72
C. April 1st of the year following year when he turns age 72 or actually retires

A

ANSWER: B. As in other IRA-type arrangements, for SIMPLE plans, the RBD is always the April 1st of the year following the calendar year in which the covered participant attains age 72. Although he continues to work for the employer providing the SIMPLE plan, Brad may not delay his first distributions beyond that date.

How well did you know this?
1
Not at all
2
3
4
5
Perfectly
39
Q

CONCEPT: MECs
QUESTION: Joan purchased a single premium whole life insurance policy in 2005. She paid one $30,000 premium for the coverage. The policy’s death benefit is $100,000. Today, the contract is worth $50,000 represented by $40,000 guaranteed cash value and $10,000 of dividend cash value. If Joan takes a policy loan of $30,000, which of the following is true?

A. She can receive the $10,000 of dividends tax-free, and $20,000 will be subject to ordinary income tax plus a 10% penalty.
B. She can borrow $20,000 tax-free, but $10,000 will be subject to ordinary income taxes plus a 10% penalty.
C. $20,000 will be subject to ordinary income tax plus a 10% penalty, and $10,000 will be a tax-free return of basis.
D. The entire $30,000 will be subject to ordinary income tax plus a 10% penalty.

A

ANSWER: C. A single premium policy purchased after 1988 is a MEC. Current CV $50,000 - basis $30,000 = $20,000 gain. Dividends in a MEC become taxable when borrowed or withdrawn. Joan’s gain is $20,000; $10,000 is her true basis in the policy. Dividends under a MEC are generally taxed. Only the gain is subject to the 10% penalty in addition to tax at ordinary rates.

How well did you know this?
1
Not at all
2
3
4
5
Perfectly
40
Q

Due to impressive growth rates in recent years, T Max, Inc. is concerned about its exposure to the corporate AMT. The company owns four substantial life insurance policies which fund a stock redemption buy-sell agreement. The officer owners have decided to switch to a cross purchase buy-sell agreement. What can or should be done with the life insurance policies that are now owned by T Max, Inc.?

Answers:
A. Use the existing life insurance policies to fund the cross-purchase buy-sell agreement(s).
B. Retain the policies for key-person coverage.
C. Sell the policies to the insureds, if they are interested, or surrender the policies.
D. Do not worry about it since the new tax law eliminated the corporate AMT.

A

ANSWER: D. If the owners who had been covered under the corporation’s stock redemption (entity) buy –sell agreement wish, they may buy the policies on their own lives that were acquired to fund that arrangement. Since the sales of the policies will be to the insureds, there is no exposure to transfer-for-value income taxation of the death proceeds. Answer A will trigger transfer-for-value problems. In answer C, surrendering the policies is not an option as they still need the buy/sell policies. Answer D is true, the new tax law eliminated the corporate AMT.

How well did you know this?
1
Not at all
2
3
4
5
Perfectly
41
Q

CONCEPT: American Opportunity Credit and Lifetime Learning Credit.
QUESTION: Lamar and Abby Sanford, who have three teenage and pre-teen children, are confused about the differences between the American Opportunity Credit (AOC) and the Lifetime Learning Credit). How would you answer the Sanfords?

A. If you elect a full Lifetime Learning Credit you cannot claim an AOC for the same expense in the same year.
B. The maximum amount of the AOC is $2,000 plus 25% of the next $2,000 for a total of $2,500 per tax year. The maximum amount of the Lifetime learning credit is $2,000 per year.
C. The Lifetime Learning Credit is no longer available after the student turns age 30.
D. The AOC is available for both undergraduate and graduate post-secondary education.

A

ANSWER: A. Answer B is incorrect. The AOC is per eligible student, per year. If a family has three children in college in the same year, each is eligible for the full AOC. Lifetime is maximum per year. The AOC is available only for the first four years of post-secondary education. No age limit applies to the Lifetime Learning Credit.

How well did you know this?
1
Not at all
2
3
4
5
Perfectly
42
Q

CONCEPT: NPV
QUESTION: Your client Bob made an investment in a commercial property. Given the risk this investment carries, he had a required rate of return of 10%. He recently sold the property and has asked you whether the investment was profitable. Was it?

A. Yes, the investment was profitable, but only if the NPV of its cash flows was positive
B. Yes, the investment would meet Bob’s required rate of return if the NPV was zero.
C. No, the investment would not be profitable if its NPV was negative
D. Yes, the investment was profitable presuming that its NPV was greater than Bob’s 10% required rate of return.

A

ANSWER: B. When, given the cash flows of an investment its NPV is zero, the investment met the buyer’s required rate of return. An investment can be profitable even its NPV is negative. A positive or a negative NPV tells whether the client achieved his required rate of return, but not the amount of the profit.

How well did you know this?
1
Not at all
2
3
4
5
Perfectly
43
Q

CONCEPT: Gross Estate
QUESTION: Your client Bob Brubaker died three months ago. Which of the following items must be included in his gross estate for federal estate tax purposes?

A. A life insurance policy owned by Bob’s daughter, Roberta. She is the beneficiary of the policy which has a death benefit of $10 million
B. $6 million of taxable gifts that Bob made to others during his lifetime
C. A general power of appointment Bob had on his deceased mother’s trust
D. Stocks and bonds gifted to his ex-wife who is a resident alien

A

ANSWER: C. Property subject to general powers of appointment is included in the gross estate of the holder of that power. The life insurance policy is owned by Roberta. The taxable gifts are added to the taxable estate. Transfers to Bob’s ex –wife may or may not be taxable gifts however, they are added to the taxable estate rather than included in the gross estate.

How well did you know this?
1
Not at all
2
3
4
5
Perfectly
44
Q

CONCEPT: MEC
QUESTION: Which of the following are amounts received by the owner of a life insurance policy would be treated as income-first distributions under a MEC contract?
I. Cash dividends
II. Interest accrued on a policy loan (added to the loan balance)
III. Dividends retained by the insurer to purchase “paid-up” additions
IV. Dividends retained by the insurer as principal or interest to pay off a policy loan

A. All of the above
B. I, II, IV
C. II, III
D. III, IV

A

ANSWER: B. When policy dividends from modified endowment contracts (MECs) are used like cash, they are distributed under FIFO rules such as those applicable to annuity distributions. Cash distributions, and dividends used to pay off policy loans are considered to be income first distributions. The interest accruing on a policy loan from a MEC is treated the same way for federal tax purposes. Dividends used to buy paid up (permanent) additions are not treated as income-first distributions. The loan interest was not distributed: It was added to the outstanding loan balance.

How well did you know this?
1
Not at all
2
3
4
5
Perfectly
45
Q

CONCEPT: Trust Types
QUESTION: Both of Rusty Whitman’s parents recently died in an auto accident. Rusty is 12 years old. Before their death, the Whitman’s had established various trusts including revocable living trusts (each parent), an irrevocable life insurance trust, and a 2503(c) minor’s trust. While Rusty is younger than age 21, the trust earnings will support his living needs. Into which of the following trusts will Rusty’s parents’ assets ultimately pass?

A. A revocable living trust
B. A 2503(c) trust
C. A Crummey trust
D. A Standby trust
E. A Family trust

A

ANSWER: E. The revocable trust will become irrevocable and will operate as a family trust for Rusty’s benefit thereafter. It is rare that a revocable trust would name its beneficiary as a 2503(c) children’s trust. 2503(c) trusts generally terminate when the minor beneficiary turns age 21.

How well did you know this?
1
Not at all
2
3
4
5
Perfectly
46
Q

CONCEPT: ADV
QUESTION: SEC registered advisers with AUM at least $100 million – are required to file annual updates to their ADV within _____ days of the end of their fiscal year.

A. 30
B. 60
C. 90
D. The number of days following the end of the adviser’s fiscal year depends on the state in which the firm maintains its office.

A

ANSWER: C. Federal covered advisers must update their ADV forms no later than 90 days following the close of their fiscal year. Smaller advisers that have registered with states securities regulators will comply with state specific deadlines for annual updates to their ADVs or equivalents.

How well did you know this?
1
Not at all
2
3
4
5
Perfectly
47
Q

CONCEPT: Gross Estate
QUESTION: George and Linda gifted $106,000 to an irrevocable living trust that includes Crummey provisions. The trust has named their two twin nephews, Larry and Barry as its beneficiaries. George and Linda consent to gift splitting. When George or Linda dies, how much, if any, of the gift will be brought back into their gross estate(s)?

A. -0-
B. $23,000
C. $46,000
D. $106,000

A

ANSWER: A. A gift of cash to an irrevocable living trust is irrevocable and removed from their gross estates for federal estate tax purposes. While both George and Linda will have made taxable gifts of $21,000, that is not reflected in their gross estate (s). Rather, it is added to their taxable estate(s) to arrive at the tax base. ($106,000 - [$64,000 ($16,000 x 4)] = $42,000. 42,000 / 2 = $21,000 each.

How well did you know this?
1
Not at all
2
3
4
5
Perfectly
48
Q

CONCEPT: Section 121 Exchange
QUESTION: Mr. and Mrs. Iverson, who file their federal income taxes jointly, sold their home for a $400,000 gain. Under Section 121, they were eligible to exclude the entire gain from their gross income. They used the proceeds of the (first) home sale to purchase another home. Now, a year and a half later, they sold the second home for a $50,000 gain. How much of the gain must they report for federal income tax purposes?

A. -0-
B. $12,500
C. $37,500
D. $50,000
E. $450,000

A

ANSWER: D. The Section 121 exclusion may be elected every two years. The Iversons only owned their second home for a year and a half. Since the question did not indicate that the Iversons sold their second home because of a job change, divorce, or unforeseen circumstance, they could not prorate the exclusion amount.

How well did you know this?
1
Not at all
2
3
4
5
Perfectly
49
Q

CONCEPT: Time-Weighted vs Dollar-Weighed Return
QUESTION: Which of the following statements is true regarding differences between time-weighted return and dollar-weighted return?

A. The time-weighted return and the dollar-weighted return are essentially different words that express the same calculation: They are Interchangeable.
B. The reason to calculate the time-weighted return rather than the dollar-weighted return is to evaluate the performance of the portfolio manager.
C. The reason to calculate the dollar-weighted return rather than the time-weighted return is to evaluate the performance of the portfolio manager.
D. The reason for calculating the time-weighted return rather than the dollar-weighted return is to identify the strategy that would eliminate reinvestment rate risk in the portfolio.

A

ANSWER: B. The time-weighted return is the geometric mean calculation. The dollar-weighted return is the IRR.

How well did you know this?
1
Not at all
2
3
4
5
Perfectly
50
Q

CONCEPT: Disability Insurance
QUESTION: Your client, Tom Adkins is concerned about his disability income insurance policy. When he bought the policy, the agent told him the company would not change the premium at any time. What should he look for in the policy provision?

A. Loss of income benefit determination
B. Guaranteed renewability
C. Own occupation definition of total disability
D. Non-cancelable continuation
E. Unilateral contract application

A

ANSWER: D. Non-cancellable policies provide premium rates that are guaranteed not to increase in the future. Insurance is a unilateral contract, which means that the insured pays the premium to the insurance company to cover the perils, but that does not answer this question.

How well did you know this?
1
Not at all
2
3
4
5
Perfectly
51
Q

CONCEPT: Profit-sharing plans
QUESTION: Which of the following statements is (are) true about profit-sharing plans?

I. They may be integrated with Social Security.
II. They may receive contributions for individual employees in excess of 25% of that participant’s eligible compensation.
III. They generally permit the employer to make flexible contributions.
IV. They may be age weighted.

A. I, II, III, IV
B. I, II, III
C. I, III
D. I, IV
E. III and IV

A

ANSWER: A. Qualified profit-sharing plans feature flexible employer contributions. They may be age weighted and integrated with Social Security. While the employer may contribute and deduct up to 25% of its overall payroll, the contribution for an individual participant may exceed that percentage.

How well did you know this?
1
Not at all
2
3
4
5
Perfectly
52
Q

CONCEPT: Insurance
QUESTION: Mrs. Roberts, age 60, is about to purchase a life insurance policy for estate planning purposes. She wants you to help her determine which policy she should buy. Various life insurance agents have proposed that she acquire one of the following policies:
* A whole life policy with Insurance Company A. The illustration shows premiums are paid in full with dividends in 10 years. (They vanish.)
* Universal life policy with Insurance Company B. The illustration shows the death benefit will be zero at age 95 based on current interest assumptions
* Whole life policy with Insurance Company C. The illustration shows premiums are paid in full with dividends in 15 years. (They vanish.)
On what criteria would you base your advice to Mrs. Roberts?
I. Review the insurance company’s ratings (A. M. Best etc.)
II. Review the size of the insurance company.
III. Review the insurance company’s past history of paying claims and its future ability to pay.
IV. Review the insurance agents’ competence, knowledge, and reliability.
V. Review whether the whole life insurance premiums will vanish and whether the universal life policy will pay the death benefit to the insured’s age 95

A. All of the above
B. I, III, IV, V
C. I, III, IV
D. III, IV, V
E. II, V

A

ANSWER: B. High ratings from two or three rating agencies should indicate that the insurance company is well able to pay its claims. The size of an insurance company is not necessarily an indicator of its financial strength. The experience and reliability of the insurance agent matters. The NAIC prohibits language indicating that the whole life premiums will vanish or the UL will last to age 95. In fact, the NAIC prohibits such language.

How well did you know this?
1
Not at all
2
3
4
5
Perfectly
53
Q

CONCEPT: Deductions to determine AGI
QUESTION: Chris Towns is the self-employed owner of the Chris Craft Stores. He reports profits or losses from his business on Schedule C. Chris Craft Stores has 10 employees. The business is very successful. As a result, the business is able to fund various employee benefits for Chris and his employees. Which of the following current benefits are either added to the front of Chris’s 1040 as part of gross income or are shown as deductions on the front of the 1040 to determine AGI?

I. Group life insurance having a death benefit of $200,000 under which Chris is the named insured
II. A SEP contribution for Chris
III. Net profit from Chris Craft Stores of $350,000
IV. A spousal IRA contribution by Chris to his wife’s account
V. Group health insurance premiums himself and his wife

A. All of the above.
B. I, II, III, V
C. I, II, III
D. II, V
E. III and IV

A

ANSWER: B. Because the face value of the group life insurance exceeds $50,000, the premium on the face value exceeding that amount is treated as compensation and becomes part of Chris’s gross income. The SEP contribution and the health insurance premiums for Chris and his wife are deductions for AGI (above the line deductions). The net profit from this unincorporated business represents income to Chris: It is part of his gross income. Chris’s income, which clearly exceeds phaseout limits, makes his wife ineligible for the spousal IRA deduction.

How well did you know this?
1
Not at all
2
3
4
5
Perfectly
54
Q

CONCEPT: ESOP Taxation
QUESTION: Bill has retired. His company provided an ESOP. Stock with a basis of $50,000 was contributed to Bob’s ESOP account. At Bill’s retirement stock having a market value of $125,000 was distributed to him. Six months after retirement, Bill sold all the shares for $150,000. Which statement below best Bill’s tax situation?

A. $50,000 was taxed as ordinary income when Bill retired; $75,000 will be taxed at LTCG rates at the time of sale, and $25,000 will be taxed at STCG rates when Bill sells the stock.
B. $150,000 will be taxed at LTCG rates when Bill sells the stock.
C. $100,000 will be taxed at LTCG rates when Bill sells the stock.
D. $50,000 was taxed as ordinary income when Bill retired; $100,000 will be taxed at LTCG rates when Bill sells the stock.

A

ANSWER: A. The unrealized appreciation is taxable as long-term capital gain to Bill when the ESOP shares are sold, even if they sold immediately. If Bill holds the shares for a period of time after distribution, any additional gain (above the net unrealized appreciation) is taxed as long or short-term capital gain, depending on the holding period. The $25,000 gain above the distribution price of $125,000) and the sale price of $150,000 is STCG because Bill held the shares for only 6 months after they were distributed to him.

How well did you know this?
1
Not at all
2
3
4
5
Perfectly
55
Q

CONCEPT: Retirement Plan Types
QUESTION: An employee contribution to which of the following plans is not subject to FICA and FUTA taxes?

A. Profit sharing 401(k)
B. SIMPLE IRA
C. SARSEP
D. 403(b)
E. Section 125 plan

A

ANSWER: E. A 125 is a flexible spending account (FSA) into which contributions are elected before the employee compensation is actually earned. All the other plans shown require FICA and FUTA tax on employee deferrals.

How well did you know this?
1
Not at all
2
3
4
5
Perfectly
56
Q

CONCEPT: Basis on inherited stock
QUESTION: Several years ago, Mrs. Pike purchased XYZ stock for $100,000. The stock was such a good investment that she bought additional shares from year to year. The additional purchases were for $10,000, $25,000, $35,000, and $20,000, respectively. On the day when Mrs. Pike died, the stock had an FMV of $500,000. Mrs. Pike’s grandson, Paul inherited the stock. He then invested his entire yearly performance bonus of $15,000 in the same stock. Six months later, he has decided to sell all the stock. What will be the taxable event if the stock Mrs. Pike bequeathed to Paul has an FMV of $600,000 and the stock that he purchased has a FMV of $25,000?

A. $110,000 of long-term capital gains
B. $410,000 of long-term capital gains and $10,000 of short-term capital gains
C. $110,000 of short-term capital gains
D. $100,000 of long-term capital gains and $10,000 of short-term capital gains

A

ANSWER: D. Paul’s gain from the sale of the stock bought with his performance bonus produces a short-term capital gain of $10,000. The gain on the amount of the stock that Paul inherited is LTCG.

How well did you know this?
1
Not at all
2
3
4
5
Perfectly
57
Q

CONCEPT: Transfer for value, gross estate
Former college roommates Barbara and Kathy are 50/50 owners of BK, Inc. When they started BK, the corporation acquired two $1,000,000 face value key-person term life insurance policies. Barbara is the named insured in one policy and Kathy in the other. Over the years, the company has paid $5,000 in premiums on Barbara’s policy and $4,000 in premium on Kathy’s policy. Barbara and Kathy have decided to use the policies to back a new cross purchase buy-sell agreement. If Barbara buys the policy in which Kathy is the named insured paid by BK, what will be the tax outcome if Kathy dies within one year following Barbara’s acquisition of the policy?

A. Barbara will receive $1,000,000 income tax-free (term insurance exclusion).
B. The policy will be subject to the transfer for value rule making the death benefit net of basis subject to federal income tax.
C. The policy will be included in Kathy’s gross estate for federal estate tax purposes (3-year rule).
D. The policy’s death proceeds will increase the fair market value of BK, Inc. by $1,000,000.

A

ANSWER: B. The policy was sold to someone other than the insured or to a business in which the buyer is an owner or partner. Whether the policy is term or permanent insurance is immaterial to the rule. Transfer for value rules makes the policy’s death benefits income taxable to the beneficiary to the extent that it exceeds basis. The 3-year rule doesn’t apply when a sale (transfer for value occurs).

How well did you know this?
1
Not at all
2
3
4
5
Perfectly
58
Q

CONCEPT: Earned Income Sources
QUESTION: Millie Tilley has the following income. How much of it would be treated as earned income for federal income tax purposes?

I. $50,000 in wages from Plant Parenthood, an S corporation Milllie works for Plant Parenthood as a landscaper.
II. $5,000 in dividends from stock held in Millie’s investment account (non-qualified)
III. K-1 income of $10,000 from an S corporation in which Millie owns 20% of the equity and is and an active participant in the business
IV. Proceeds from the sale of an oil painting inherited from her great aunt that generated a $5,000 long-term capital gain

A. $70,000
B. $65,000
C. $60,000
D. $55,000
E. $50,000

A

ANSWER: E. Only Millie’s salary would be classified as earned income. The other answers indicate investment income which is, by nature unearned. K-1 from an S corporation represents a distribution of profits and thus, is treated as investment income. Although Millie is an active participant in the S corporation’s activity, this is true.

How well did you know this?
1
Not at all
2
3
4
5
Perfectly
59
Q

CONCEPT: Sale for value
QUESTION: Joan Thomas sells her term life insurance which has a face value of $200,000 policy to Linda Bell for $1,000. Joan dies five years later. Linda, who immediately named herself as beneficiary of the policy, paid $4,000 in premiums over the past five years. To what extent, if any, will the insurance policy proceeds be subject to federal income tax?

A. None of the death proceeds will be subject to federal income tax because term life insurance policy does not trigger transfer for value rules.
B. $199,000 will be taxable to Linda for federal income tax purposes.
C. $195,000 will be taxable to Linda for federal income tax purposes.
D. The $200,000 face value of the policy will be included in Joan’s gross estate for federal estate tax purposes.

A

ANSWER: C. Her payments increase her basis. When Joan sold the life insurance policy to Linda, that sale triggered transfer for value rules making the death proceeds in excess of basis taxable to the beneficiary who is now Linda. Linda will receive the death proceeds subject to federal income tax. When Joan dies, nothing attributable to the life insurance policy will be included in her gross estate. The 3-year throwback rule does not apply when a life insurance policy is sold.

How well did you know this?
1
Not at all
2
3
4
5
Perfectly
60
Q

CONCEPT: UTMA
QUESTION: This year, Joe Jackson started a 529 college savings plan for his sister’s son, Jimmy. He gifted $75,000 ($15,000 for five years) to a 529 plan that is operated by the state in which both he and Jimmy reside. However, Joe does not feel that the ending balance in the 529 account will be enough to pay for his nephew’s total college costs. Joe has observed that his sister and brother-in-law seem to live well beyond their means. His brother-in-law is an executive earning $500,000 per year. His sister and her husband travel extensively to compete in winter sports. They have not earmarked any money for their son’s education. Joe’s financial advisor said he could fund an UTMA with $50,000 and invest it in AA rated nationally diversified municipal bonds that would generate around $1,000 in annual income. The UTMA would increase by the interest on a tax-free basis and all the funds can be distributed Jimmy’s for college expenses. Joe could name himself as the custodian. How would you, a CFP ® practitioner respond if Joe asked you whether or not this advice is sound?

A. Advice should be implemented
B. The gift to the custodial account seems reasonable, but it would be treated as a taxable gift for federal gift tax purposes
C. He has already used up all of his annual exclusions for the next five years
D. The gift would have to be made first to his brother-in-law who would then in turn contributed to Jimmy’s UTMA.

A

ANSWER: A. Both B and C are true statements, but answer A is the best planning. The UTMA invested in municipal securities appears to be sensible while the transfer of the $50,000 to Jimmy’s custodial account is a taxable gift, Uncle Joe can use his gift tax property exemption of $12,060,000 to avoid current gift tax. Under kiddie tax rules, earnings generated by the UTMA may be taxed at 37% plus the 3.8% Medicare investments tax. In that light tax-free interest from municipal bonds make sense. The tax-free income should enable the account to have a FMV of $65,000 in 10 years.

How well did you know this?
1
Not at all
2
3
4
5
Perfectly
61
Q

CONCEPT: Income Tax Rules
QUESTION: Mrs. Tilden, a widow, has gifted extensively to her daughter, Sally. She used her entire gift property exemption amount and actually paid federal gift tax on her most recent gifts. Mrs. Tilden recently married Bill Widner. She is considering gifting him $1,000,000 with the written understanding that he will then gift the $1,000,000 to Sally. How would you respond after she explains her strategy?

A. This is an effective way to accomplish effective gift tax planning
B. The transfer of $1,000,000, reduced by one annual gift tax exclusion from Mrs. Tilden to Bill Widner will be a taxable gift.
C. Mrs. Tilden and Bill Widner need to be married for one year for this technique to work.
D. The transfer of $1,000,000 reduced by one annual gift tax exclusion from Mrs. Tilden to Bill Widner will be a non-taxable gift.

A

ANSWER: B. It appears that the gift will not qualify for the marital deduction. To qualify for the deduction, the donee spouse must be given the property outright or must have at least a right to the income from the property and a general power of appointment over the principal. The IRS would consider this to be a step transaction and thus a fraudulent transfer.

How well did you know this?
1
Not at all
2
3
4
5
Perfectly
62
Q

CONCEPT: Retirement Plan Type
QUESTION: Law school classmates, Mr. Ball and Mr. Desmond are both attorneys. At this time, their law practice which specializes in personal injury law, BaDe, PC (Professional corporation) does not provide any retirement plan. The two Counselors want to establish a qualified plan. They want a plan that requires no employee contributions nor mandatory annual employer contributions. BaDe currently has only two other employees who are part-time paralegals. The practice plans to hire another attorney. Which of the following plans would you recommend for BaDe, Attorneys at Law?

A. Given the current cash flow of BaDe and the objectives of its owners, no plan is recommended at this time.
B. A SEP
C. A Profit-sharing plan
D. A Profit-sharing 401(k) plan
E. A Defined-benefit plan

A

ANSWER: The profit-sharing plan best fits the objectives and preferences of Esquires Ball and Desmond. They do not want a plan that requires mandatory annual funding. The defined-benefit plan would not fit their model. The attorneys do not want to defer personal income. This eliminates the 401(k). The SEP is not a qualified plan. The SEP would probably have to cover part-time employees. Relative to the profit-sharing arrangement, the ERISA 1,000-hour rule would exclude the part-time employees from participating.

How well did you know this?
1
Not at all
2
3
4
5
Perfectly
63
Q

CONCEPT: Wash Sale Rule
QUESTION: Your client, Mr. Smith purchased and sold the following stocks during a two-year period. Which situation or situations below created a “wash” sale?
NOTE: All transactions are 100 share round lots.
I. March 1st purchased ABC @ $15; December 1st purchased ABC @ $10; December 31st sold ABC @ $10
II. November 30th purchased LMN @ $50; December 15th purchased LMN @ $52; December 29th sold LMN @ $54
III. January 1st purchased XYZ @ $60; February 15th sold XYZ @ $50; March 16th purchased XYZ at $52

A. I, II, III
B. I, III
C. I and II
D. II
E. III

A

ANSWER: C. The ABC and the XYZ transactions violate the wash sale rules
–ABC’s basis is $10 cost plus $5 recognized loss. (December 1st and December 31st)
–XYZ’s basis is $52 cost plus $10 recognized loss. (February 15 and March 16) There are only 28/29 days in February.
-LMN was sold for a gain.
The deduction is disallowed for any loss from any sale or other disposition of stock within a period beginning 30 days before and ending within 30 days after the date of the sale or disposition. A gain cannot create a wash sale.

How well did you know this?
1
Not at all
2
3
4
5
Perfectly
64
Q

CONCEPT: Social Security
QUESTION: Hal, age 63, is trying to decide whether he should begin taking Social Security benefit 36 months before his FRA. He is a fully insured worker. Regarding Hal’s situation, which of the following statements is correct?

I. Once Hal begins taking Social Security Retirement benefits, he will be eligible for Medicare.
II. Hal will receive 80% of his PIA that would apply at his FRA
III. If Hal works part-time, his benefits will be reduced by 20%.
IV. If Hal works part-time, his benefits will be reduced $1 for every $3 he earns above a specific earnings threshold.
V. If Hal does not work, his benefits may or may not be subject to federal income taxation.

A. I, II, III, IV
B. II, IV, V
C. III, IV
D. II, V
E. IV, and V

A

ANSWER: D. Hal will not be eligible for Medicare until age 65 (Answer I). Claiming benefits 36 months early would result in a 20% permanent reduction in Hal’s benefits (36/180). If Hal works part-time, before the year in which he reaches his full retirement age (FRA) his benefits will be reduced $1 for every $2 he earns above a specific threshold. During his FRA year only he will lose $1 in benefits for every $3 by which his earned income exceeds that threshold. Hal’s Social Security retirement benefits will be subject to federal income tax if his AGI plus ½ of his benefits (provisional income) exceed $25,000+. Given that we do not know Hal’s provisional income indeed his Social Security benefits may (or may not) be subject to federal income tax.

How well did you know this?
1
Not at all
2
3
4
5
Perfectly
65
Q

CONCEPT: Corporate Annual Reports
QUESTION: Corporate annual reports would generally not include which of the following?

A. Depreciation methods
B. Stock options
C. Profitability projections
D. Inventory methods
E. Outlook for the firm’s products in various industries in which it operates

A

ANSWER: C. Profitability projections should not be included in corporate annual reports. The SEC believes that such projections could mislead shareholders and others.

How well did you know this?
1
Not at all
2
3
4
5
Perfectly
66
Q

CONCEPT: Prepaid tuition plans
QUESTION: Mr. and Mrs. Grandparent have paid $50,000 into their granddaughter’s “prepaid tuition program.” The arrangement qualifies as a Section 529 program. Which of the following is true?

A. Prepaid tuition plans may only pay for tuition and mandatory fees.
B. Prepaid tuition plans do not affect the expected family contribution for determining available financial aid.
C. Prepaid tuition plans do not allow the beneficiary to attend a private or out-of-state college.
D. Prepaid tuition plans provide a rate of return that is linked directly to the return on the securities in which the prepaid tuition plan invests.
E. If the qualified higher education expenses (QHEE) are less than the total distributions, the distributions from the prepaid plan are then treated as taxable income.

A

ANSWER: A. If the beneficiary of a prepaid tuition (529) plan attends a private or out-of-state college, the program will determine the value of the contract. If the QHEEs are equal to, or greater than, the total distribution, they are tax-free. Prepaid tuition plans pay for tuition and mandatory fees. They do not cover room and board. Prepaid tuition plans do affect the expected family contributions.

How well did you know this?
1
Not at all
2
3
4
5
Perfectly
67
Q

CONCEPT: ERISA
QUESTION: Toby, age 72, has been taking distributions from his qualified plan. Now, to his dismay, he has been sued and lost in court. His only real asset is the qualified plan distributions. Can the plaintiff collect against his qualified plan distributions?

A. No, ERISA forbids “alienation of plan benefits.”
B. No, Toby can file for Chapter 7 bankruptcy and keep his plan benefits.
C. Yes, but only for a portion of the distributions
D. Yes, because that is his only asset

A

ANSWER: C. The anti-alienation provision only applies during the accumulation period, not after the account goes into pay status. Once the distributions are paid out, they become an asset of the participant, subject to any creditor’s claims that may be pending when the benefits are received.

How well did you know this?
1
Not at all
2
3
4
5
Perfectly
68
Q

CONCEPT: Trust Taxation
QUESTION: Mark Spout created an irrevocable trust for the benefit of his dependent children. Mark named the local bank as trustee of the trust and authorized it to invest in stocks, bonds, and negotiable certificates of deposit. Included in the investment authority is the right to use trust income to purchase insurance on Mark’s life. All funds are currently invested in high-yielding bonds paying 7% semiannual interest on a par value of $100,000. Twenty-five percent of the bond investment income is being used to pay the premium on a policy on Mark’s life. Which taxpayer must pay tax on the income of the trust and why?

A. The bank because of its broad authority as trustee
B. The children because the income is paid by them
C. The trust because it is irrevocable with no benefits to grantor
D. Mark because of the grantor trust rules

A

ANSWER: D. If any portion of the trust income is, or may be, used to purchase insurance on the life of the grantor or grantor’s spouse, then the trust is a grantor trust. The trust is/was also used to benefit his dependent children (support). 25% is to purchase insurance and the remainder is used for support.

How well did you know this?
1
Not at all
2
3
4
5
Perfectly
69
Q

CONCEPT: Trusts
QUESTION: Scott Harding died recently (at age 74), with a taxable estate of roughly $8 million. Some years ago, he had seen an estate tax attorney and completed an extensive estate plan with a variety of trusts. Among his assets is a stone-constructed New England farmhouse having a current fair market value of $2 million. Scott’s will bequeathed the home to Clarisse who, due to authoring a best-selling mystery novel, is worth approximately $20 million in her own right. Nevertheless, when Scott died, Clarisse, (under the guidance of her tax advisor), did not want to own the home outright, fearing it would become part of her already substantial gross estate. However, she wishes to live in the home where she can continue to write and enjoy her grandchildren for the rest of her life. The home had been in the Harding family since its arrival in America in 1789. At the time of his death, it was owned fee simple by Scott. There is great sentiment associated with the home: Scott and his wife, Clarisse (age 66) were married in the home and raised their two children Rebecca and Jonah (now adults) there. What technique, if any, will accomplish Clarisse’s goals?

A. Clarisse cannot have her cake and eat it too: If she disclaims ownership of the home, she may not live there without paying fair market rent.
B. With a proper provision in Scott’s will, if Clarisse disclaims ownership of the home, it can be transferred to a disclaimer trust, the terms of which permit Clarisse to live in the home for her lifetime. This would be a family trust.
C. With a proper provision in Scott’s will, if Clarisse disclaims ownership of the home, it can be transferred to a disclaimer trust, the terms of which permit Clarisse to live in the home for her lifetime. This would be a marital trust because Clarisse has a life interest in the home.
D. A qualified personal residence trust (QPRT) should accomplish Clarisse’s dual goals of not including Scott’s home in her gross estate and the right to live in the home for her entire lifetime.

A

ANSWER: B. Many well written wills include disclaimer trust provisions which give the surviving spouse the ability to put specific disclaimed assets into the trust by disclaiming ownership of a portion of the estate. Disclaimed property interests are transferred to the trust, without being taxed.

Provisions can be written into the trust that provide for regular payouts from the trust to support survivors, or in the case of the Hardings, the right to occupy (but not own) property. The trust can also be written so that surviving minor children can also be provided for, as long as the surviving spouse elects to disclaim inherited assets, passing them on to the trust. To keep the assets from being included in Clarisse’s estate, the trust would have to be a family trust rather than a marital trust.

If a disclaimer trust is used, the full extent of the tax planning occurs upon the death of the first spouse. At that point, the surviving spouse can either accept the trust assets or disclaim them. I f he or she disclaims them into the disclaimer trust, the trust will function like a credit shelter trust that will shelter the assets from inclusion in the surviving spouse’s estate. But if there is no tax reason to use credit shelter planning, the spouse can simply receive the assets outright. This allows tax-planning flexibility without creating unnecessary complication. Using answer D will mean for the life of the QPRT it could be brought back into her estate. It is not a bad answer. With Answer B, the house uses the exemption.

70
Q

CONCEPT: Financial Planning Process
QUESTION: Your client, Martha, requested a meeting with you. Martha is married to Glen. Glen is a workaholic. You only met him once to set up their joint account. Martha always makes all the investment decisions. Glen makes a lot of money, but Martha saves a majority of it through frugal living. As the meeting starts, Martha is quite blunt. Glen has taken up with one of the girls at the plant. Cash flow to Martha from Glen has ceased. He set up a separate bank account in his name. She wants your advice on how to handle the assets in the investment account.

A. Tell her to break the joint tenancy account and retitle as tenants in common.
B. Tell her you need to secure statements from both her and Glen to proceed with any changes.
C. Tell her to see a divorce attorney before you can proceed to change the account into her name.
D. Call up Glen and ask him to come into your office.

A

ANSWER: B. The account is in joint names, both are clients. She never says she wants to divorce him. Also, to do Answer C, she has to get a divorce before you can proceed with the account titling. Do you have enough of a relationship with Glen to select Answer D? Doubtful. Answer A depends on Answer B.

71
Q

CONCEPT: Financial Planning Process
QUESTION: An existing client calls you, a CFP® professional, with a question about a topic you aren’t very familiar with. She tells you her husband served in the military decades ago and is now receiving care in a nursing home. She read a brochure at this new facility about a VA benefit called Aid and Attendance that could pay her around $1,200 per month as the spouse of a serviceman who is receiving skilled nursing care. She would like you to help her qualify for the benefit.

You tell her that you will do some homework and arrange a meeting with her the following week to discuss their situation further. After doing some research, you discover that there are very stringent income and asset restrictions to qualify for benefits. Since they have been clients for years, you know they have a net worth of around four million dollars. In your meeting you should:

A. Explain the income and asset thresholds that are well lower than their current situation. Help her understand that this benefit is not intended for people in their financial position and recommend they not pursue the benefit further.
B. Tell your client you aren’t an expert in this matter and put her in touch with an attorney with whom you have worked the past whom you know to be a military veteran.
C. Review options for how the client may effectively reduce income and gift assets to qualify for the benefit.
D. Recommend she purchase long term care insurance and review their current retirement and investment objectives for other options to meet their needs.

A

ANSWER: C. In D, you never address her objective and discuss options on how to qualify; you only give recommendations apart from it. A is a good answer however you should let her determine if she is willing to go to lengths to qualify, you shouldn’t be making the decision for her by dismissing the option. B would be a great answer, but it is unclear if the attorney is competent in the subject. In this meeting, you should review the client’s options as she requested, answer C. Once that is done you may then decide to advise a more suitable solution (Answer D). Subjective. If you answered B, I am not saying you are wrong. This is a practice question.

72
Q

CONCEPT: Life Insurance Surrender
QUESTION: A corporation purchased a whole life insurance policy for the life of a key employee. The corporation was both the owner and beneficiary under the contract. The corporation did not deduct the premium but instead listed the premiums paid as an asset on its financial statement. The corporation borrowed from the insurance company and various banks using the policy as collateral for the loans. Today, the corporation surrendered the policy for a gain. How will the gain be treated for income tax purposes?

A. As a long-term gain because a termination of a right with respect to property is a capital asset of the taxpayer
B. As a long-term gain because life insurance contracts are considered capital assets
C. As a long-term gain because the cash value growth was not treated as ordinary income, but it is treated as a capital asset
D. As ordinary income
E. As a return of basis, a nontaxable event

A

ANSWER: Life insurance surrender gain above basis is always taxed as ordinary income.

73
Q

CONCEPT: Required Rate of Return
QUESTION: When dealing with a client’s estimated unequal cash flows from a potential investment, what is the major difficulty that you, as a financial planner, may encounter?

A. Whether to use time value of money
B. What discount rate to use
C. Whether the investment under consideration should be rejected if the NPV is a negative number
D. How the IRR will be reinvested

A

QUESTION: B. Answer C is a good answer. The difficulty is in determining what discount rate (client’s required rate of return) to use for the investment.

74
Q

CONCEPT: Charitable Deduction
QUESTION: Mr. Lukes owns a convenience store. After a storm hit his city, he donated $1,000 worth of food to the American Red Cross. He feels he could have sold the food for $2,000. Normally, the expenses associated with selling the food consume 50% of his profit. What amount of charitable deduction can he take?

A. No contribution deduction is allowed.
B. 50% of his AGI
C. $1,000
D. $2,000

A

ANSWER: C. For self-employed individuals, partnerships, and Subchapter S corporations, the contribution amount for inventory must be reduced by the amount, which would have been recognized as gain if the property had been sold by the donor at its fair market value at the time of its contribution to the charitable organization. In other words, the charitable contribution of inventory is limited to its cost.

75
Q

CONCEPT: Annuitization
QUESTION: Harry Potter bought a $25,000 single premium deferred annuity 30 years ago at age 30. Now approaching retirement, he is trying to decide if he should take the cash value ($110,000) as a lump sum or annuitize the policy over a 20-year single life expectancy of $725 per month. When he enters his retirement years his tax bracket will drop to 12% and he is concerned about income. What would you recommend if he feels he can invest the lump sum and achieve a 6% after-tax return?

A. He should take a lump sum and pay the tax
B. If he takes a lump sum, he will have to pay the tax plus a 10% penalty (annuity rules), he should annuitize.
C. He should annuitize the contract.
D. He should take a lump sum because all he will get is 20 years of payments.

A

ANSWER: C. Well, if he takes a lump sum Answer A he will pay 12% on $85,000 ($110,000 - 25,000 basis) or $10,200. ($110,000 -10,200) x 6% = $5,988/year. $725 x (1 - 12.00%) = $638.00, $638.00 x 12 = $7,656/year. Answer C has a higher net of tax payout. Answer B is wrong. He is over 59½ so there is no 10% penalty. Answer D is wrong as the payments continue after 20 years but are 100% taxable.

76
Q

CONCEPT: Taxation of Death Benefits
QUESTION: Tommy Todd died this year. He was an employee of a large company. The company owned a group life policy covering him for $50,000 and the company also had a company benefit of paying a $5,000 death benefit outright. If both death benefits were paid to Tommy’s wife, how much would be subject to income tax?

A. $0
B. $5,000
C. $50,000
D. $55,000

A

ANSWER: B. A group life policy is normally owned by the company. The company pays the premium, but the benefits (up to $50,000) are tax-free. The $5,000 death benefit paid the company used to be tax-free (a De Minimis fringe benefit) but is now taxable.

77
Q

CONCEPT: Depreciation vs Improvement
QUESTION: During the year, Fred Smith had the following expenditure for his rental house. The expenditures are listed below. Fred has a full-time job and files a Schedule E (active participation) for the rental house. Which of the expenditures must be depreciated rather than deducted as an expense on his Schedule E?

I. Replaced a screen in a window
II. Replaced the air conditioning system
III. Built a swimming pool
IV. Installed a new water heater
V. Hired a lawn service to cut the grass

A. I, II, V
B. I, IV, V
C. II, III, IV
D. III, IV
E. III

A

ANSWER: C. Items I and V are expenses. The others are improvements even though II says replace and IV says install. There is no list of what are expenses and what are improvements.

78
Q

CONCEPT: 1244 Exchange
QUESTION: Gail Goodrich, single, owned a small corporation, GG, Inc. Due to business reversals, she had to close the business, and the stock became worthless. Which of the following is true if the loss is $120,000?

A. If she meets the requirements of Section 1244, she can take an ordinary loss of $100,000 and a capital loss of $3,000. The remaining $17,000 is a carry-forward loss.
B. She can take a $103,000 short-term loss.
C. She can take a $53,000 long-term loss.
D. None of the above

A

ANSWER: D. Answer A is wrong because she is single, not married. Section 1244 allows for a $100,000 ordinary loss for married filing jointly and a $50,000 ordinary loss for single persons. Answers B and C are wrong because we do not know how long she was in business. The correct answer for single is $50,000 ordinary loss plus $3,000 capital loss and a $67,000 carry forward.

79
Q

CONCEPT: Immunization
QUESTION: What are the purposes of immunizing a bond portfolio?

I. To reduce the bond portfolio’s market losses
II. To earn a specific rate of return from the bond portfolio over a given period of time, regardless of what happens to market interest rates
III. To offset interest rate risk with reinvestment rate risk
IV. To avoid buying zero coupon bonds

A. I, II, III
B. I, II, IV
C. II, III, IV
D. II, III
E. III, IV

A

ANSWER: A. If interest rates rise, interest rate risk causes the value of the bonds to drop, but the client can earn more on coupon payments that are reinvested. If interest rates decline, interest rate risk causes the value of the bonds to rise, but the client will earn less on coupon payments that are reinvested. The typical method of immunizing involves assembling and appropriately managing a diversified portfolio of bonds.

80
Q

CONCEPT: Social Security
QUESTION: Taxes are triggered for Social Security recipients when modified adjusted gross income (MAGI) exceeds specified amounts. To calculate whether Social Security benefits will be taxable, which types of income must be considered?

I. Pension distributions (RMDs)
II. Rental income (passive)
III. Section 162 Insurance Premiums
IV. Interest earned on CDs
V. Tax-exempt interest from private purpose municipal bonds

A. All of the above
B. I, II, IV, V
C. I, III, V
D. II, IV, V
E. III, IV

A

ANSWER: A. Rental income is income even though passive. Section 162 insurance premiums are charged to the employee. Tax-exempt (public or private) affects MAGI.

81
Q

CONCEPT: Insurance Coverage
QUESTION: While on a trip, Sandra’s wedding ring disappeared. The ring was worth $10,000. She has an HO-3 policy. Which one of the following statements is true?

A. The coverage on the ring is limited to $1,000.
B. The coverage on the ring is limited to dollar amount ($1,000 - 1,500) if the loss is due to theft.
C. If she has more than $10,000 personal property coverage, the ring is covered for its full value.
D. The ring is not covered.

A

ANSWER: B. The ring is only covered if its disappearance is due to theft. The dollar amount shown is not important. See Insurance pre-study lesson 3. It is a concept question (theft). There is no coverage unless a theft occurred. The theft must be reported to the police. It is a commonsense question/answer. Some policies cover jewelry up to $5,000.

82
Q

CONCEPT: Public vs Private Charities
QUESTION: Which of the following organizations would be considered public charities (50% organizations)?

I. Education organizations with regular faculty and curriculum & regularly enrolled students
II. Hospitals
III. The Rotary Club (a not-for-profit), which raises money for public causes
IV. The United Way
V. Public libraries

A. All of the above
B. I, II, III, IV
C. I, II, IV, V
D. II, IV
E. III, V

A

ANSWER: C. Rotary is a non-profit (Not-for-profit). It isn’t a public or private charity. The Rotary Club would have to obtain IRS approval to receive tax deductible contributions.

83
Q

CONCEPT: FSA
QUESTION: Which statements regarding a health FSA are correct?

I. It may receive contributions from an eligible person.
II. Allowable contributions are not subject to FICA.
III. Allowed contributions are not included in income.
IV. Reimbursements from the health FSA used for expenses are not taxed.

A. I, III
B. I, II, IV
C. I, II, III
D. II, IV
E. III, IV

A

ANSWER: C. Statement IV is wrong because it does not specify qualified medical expenses.

84
Q

CONCEPT: Investment interest
QUESTION: A client has investment income of $500,000. The client takes a loan of $50,000 from a bank to buy 20 acres of land for investment purposes. The real estate will be the collateral for the loan. The loan cost for the year is 10% or $50,000. Is the interest tax deductible?

A. No, it is a passive investment (passive loss).
B. No, the client is not materially participating in real estate.
C. Yes, up to investment income.
D. Yes, but the client has to have an AGI of at least $50,000.

A

ANSWER: C. Investment interest ($50,000) is tax deductible up to investment income ($50,000). Investment interest is any interest incurred to purchase property that is held for investment, such as raw land.

85
Q

What is the advantage between taking excessive income as a stockholder from a corporation or an S corporation if the corporation is very profitable?

A. Taking earned income as compensation from a corporation will qualify the stockholder to ultimately get more Social Security benefits.
B. Taking earned income as compensation from a corporation will qualify the stockholder for a large retirement contribution.
C. Taking limited income from an S corporation will reduce FICA and FUTA taxes because the remainder of income will be unearned income.
D. Taking limited earned income from an S corporation and the remainder as unearned income will reduce corporate taxes.

A

ANSWER: C. In answer A and B there are salary caps that limit what qualifies for Social Security benefits and retirement contributions. But compensation is subject to unlimited Medicare taxes (disadvantages). That is why Answer C is a better answer. Unearned is not subject to Medicare. Answer D is incorrect, an S corporation does not pay corporate taxes (conduit entity). You may have chosen Answer D because it is an advantage for an S corporation owner. Yes, S owners reduce their wages to pay less FICA taxes by taking the maximum unearned income. This is why many people do an S corporation - avoid FICA taxes. But that is not what Answer D says. You have to answer the question as written.

86
Q

CONCEPT: MSA
QUESTION: Which of the following is (are) true of an existing Medical Savings Account (MSA)?
I. A MSA may receive contributions from any person, including an employer or family
member, on behalf of an eligible person.
II. Contributions by the eligible individual are deductible even if that person does not
itemize.
III. Employer contributions are not included in the eligible person’s income.
IV. Both the employer and eligible person may contribute to the MSA in the same tax year.

A. I
B. II, III
C. I, II, IV
D. I, III, IV
E. All of the above

A

ANSWER: B. Statements I and IV are wrong because the only parties that can contribute are the eligible person or the employer but not both in the same year. Contributions by the eligible individual go on the front of the 1040 (adjustments to income).

87
Q

CONCEPT: Financial Planning Process
QUESTION: Judith Everet, age 55, has been your client for years. Divorced 10 years ago with generous marital settlement, very conservative with a low risk tolerance. Asset portfolio for past years since 2007 market downturn has been 70% in bonds, 20% in money market, and 10% in growth and income funds. As a result of inflation and lifestyle changes, travel to Europe and Far East, her portfolio has gone down. She has taken all the income. She comes to you today to take 40% and invest it in emerging growth. She is taking 20% from bond and all of the money market as her source of investment. What should you do?

A. Make the investment
B. Make the investment document her decision
C. Do not make the investment, explain to her that the risk is beyond her profile.
D. Do not make the investment, educate her on the risk associated with emerging growth.

A

ANSWER: D. Very tough question/answer. All the answers are good but educating her is best.

88
Q

CONCEPT: Actual Cash Value coverage
QUESTION: Your client, Susan bought a deluxe refrigerator 10 years ago. It cost Susan $1,200 when new. A current model of the same refrigerator would now cost $1,500. The appliance was expected to last 15 years. However, it was destroyed by a grease fire that started in Susan’s kitchen. How much would the insurance carrier pay if Susan’s policy provided for used actual cash value coverage?

A. $500
B. $1,000
C. $1,200
D. $1,500

A

ANSWER: A. Actual cash value is the current replacement value of $1,500 less depreciation of 10/15 of the current replacement value of $1,500 ($1,000) or $500.

89
Q

CONCEPT: Wash sale rule
QUESTION: Mr. Teig bought common stock in Company X two years ago. Unfortunately, the insider information he was given turned out to be false. The new drug produced by Company X has yet to be approved by the Food and Drug Administration (FDA). As a result, he is sitting on a large loss. He needs the loss to offset capital gains. However, he is still optimistic about the stock. What can you recommend to avoid a wash sale?

A. Sell the stock and buy a deep-in-the-money call.
B. Sell the stock and sell a deep-in-the-money put.
C. Sell the stock and have Mrs. Teig buy the shares the next day.
D. Sell the stock and wait 31 days to buy it back.

A

ANSWER: D. The question says it is a deep-in-the money call. For example, if the market value is $30, then the exercise price must be less, like $15 - 20. He will have to pay at least $10-15 to buy the option. The call and put options leave Mr. Teig in the same position as if he owned the stock. Mrs. Teig’s purchase of the stock would probably fall under the related party rules causing it to be a wash sale. Answer D is the best answer.

90
Q

CONCEPT: Investment Interest
QUESTION: Your client, Clark Crawford wants to buy stock in the Wonder Widget Company, a regular corporation. However, the stock is not publicly traded. To buy the stock, Clark takes a loan of $100,000 from a bank. The bank requires the stock to be held as collateral. Also, Clark receives substantial investment income from a large, well-diversified corporate bond portfolio. Would Clark be able to deduct the loan interest?

A. Yes, as long as he has investment income
B. Yes, because he pledged the stock for the loan
C. No, because the stock is not publicly traded stock
D. No, because Clark does not materially participate in the operations of the Wonder Widget Company, the interest is passive.

A

ANSWER: A. This is simply a form of margin or investment interest. It doesn’t matter that the stock isn’t publicly traded.

91
Q

CONCEPT: TIPS
QUESTION: Melissa purchased some Treasury Inflation-indexed securities (TIPS). She asks you, her financial planner, about the tax ramifications of the securities. Which one of the following statements is incorrect?

A. The interest is subject to federal taxation when received.
B. The inflation adjustment to principal is also subject to federal taxation in the year the adjustment is made.
C. The interest and inflation adjustment may be deferred until the bond is redeemed or maturity occurs in 30 years.
D. The deflation adjustment to principal is also subject to federal tax deduction in the year the adjustment is made.

A

ANSWER: C. Answer C is referring to I bonds. Tax reporting is similar to EE bonds. TIPS are adjusted for deflation as well as inflation. In deflation, the principal is adjusted downward and interest payments are less than they would be. This answer, as written, came from Treasury direct website.

92
Q

CONCEPT: Includable Compensation
QUESTION: Jim Harrison, President of Harrison Office Supply and Furniture, Inc. wants to establish a profit-sharing 401(k) plan. He would like to exclude some of the sales staff but is uncertain if qualified plan rules permit this. Jim would like to cover all W-2 (salaried) employees. The office staff consists of 17 W-2 employees (4 are classified as highly compensated and 3 are key employees). The warehouse staff consists of 38 W-2 employees, none of whom are highly compensated. The sales department consists of 6 1099 commissioned salespeople (all on straight commission and all have complete control over their schedule and duties.) and 1 W-2 employee. Charles Porter, the sales manager is a highly compensated key employee. Porter receives a salary, bonuses, and commissions on his own sales. Which of the following statements are correct regarding permissible coverage and includable compensation that would generally apply to the 401(k) plan that Jim is considering?

I. If the salespeople are deemed to be common-law employees, the plan may exclude them provided either the ratio percentage test or the average benefit test are passed to indicate that the plan is not discriminatory.
II. If the salespeople are independent contractors, their participation they could cause the plan to be disqualified.
III. Commissions may be excluded from includable compensation for elective and employer contribution purposes.
IV. Bonuses may be excluded from includable compensation for elective deferral and employer contribution purposes.

A. I, II
B. I, II, III
C. I, III, IV
D. All the above

A

ANSWER: D. Participation eligibility in a qualified plan is contingent on employee status (Sole-proprietors and partners may establish their own qualified plan). An Independent contractor may participate in his own qualified plan but may not participate in another entity’s qualified plan unless he is also deemed to be a regular or common law employee of that entity. It is possible to exclude commissions and bonuses from includable compensation provided that doing so does not discriminate against a non-highly compensated participant.

For example, if commissions are not counted as includable compensation and the key employees never receive commissions, the non-key employees have been discriminated against because their includable compensation is being reduced (by the amount of commissions earned) while the key employees compensation is not reduced at all. Very involved question and answer.

93
Q

CONCEPT: Financial Planning Process
QUESTION: Does the CFP Board consider an engagement to be financial planning solely because the CFP® professional used the multi-step process?

A. Yes, that is how the CFP Board defines financial planning
B. No, the CFP Board recognizes that the steps are not unique to the financial planning process
C. Yes, any dialogue between a CFP® professional and a member of the general public is deemed to be “financial planning.”
D. The CFP® Board does not address the issue.

A

ANSWER: B. The financial planning process steps may occur in connection with other activities. Examples include gathering client data as part of a suitability analysis. Question idea came from CFP® Board’s frequently asked questions. The financial planning process steps may occur in connection with other activities. Examples include gathering client data as part of a suitability analysis.

94
Q

CONCEPT: QDRO
QUESTION: Mrs. Adams is divorcing Mr. Adams. She participates in a money-purchase plan at work. Currently, her husband is the named beneficiary of the defined contribution pension. She wants to change the beneficiary to her daughter as soon as possible. The money-purchase plan will be subject to a QDRO. When will the QDRO be valid?

A. When the divorce is finalized
B. When Mr. and Mrs. Adams sign the QDRO
C. When the plan administrator has approved the QDRO that has been entered with court and signed by the judge.
D. When Mr. and Mrs. Adams separate and live-in separate houses
E. When she gets awarded the custody of her children.

A

ANSWER: C. The best practice is that a QDRO is entered with the court at the same time as the judgment of divorce and qualified by the plan administrator.

95
Q

CONCEPT: Special Use Valuation (2032A)
QUESTION: All the following are requirements for real property to qualify for and retain special use valuation (2032A) EXCEPT:

A. The decedent must have been a U.S. citizen or resident.
B. The real property seeking to be eligible for the valuation discount must pass to a qualified heir.
C. Either the decedent or a member of the decedent’s family must have materially participated in the operation of the farm or business for at least ten years immediately prior to the decedent’s death.
D. A written agreement must be filed with the estate tax return (Form 706) and must be signed by all parties having an interest in the real property.

A

ANSWER: C. Prior to death, the five-out-of-eight-years rule applies.

96
Q

CONCEPT: IRA eligibility
QUESTION: Tom Thomas, single age 62, just retired. He is taking Social Security retirement benefits early from which he is receiving $1,500 per month. He also annuitized his pension account from which he will receive lifetime income of $12,000 per month. His investments are in several dividend paying mutual funds. Tom has asked you about Roth IRAs. He knows there is no 72 rule with Roth IRAs. He is not using all his income and would like to reinvest in an account that would grow tax-free. What would you recommend?

A. He could contribute to a non-deductible IRA.
B. He cannot contribute to a Roth because his AGI is above the phaseout for single persons.
C. He could contribute to a non-qualified annuity.
D. He could roll some of his pension distributions into a Roth IRA.

A

ANSWER: C. NO EARNED INCOME. He does not have any compensation to do an IRA or Roth. Answer B is true, but it does not answer the question. He has no compensation. The pension distributions cannot be rolled into a Roth. NOTE: The question says grow tax-free. Grow makes a difference.

97
Q

CONCEPT: interpolated terminal reserve - what is that?
QUESTION: Clarice, while living, assigns the ownership of a whole life insurance policy to her niece, Clara. How would the Internal Revenue Code classify the policy’s value for transfer tax purposes?

A. As a gift
B. As the interpolated terminal reserve plus the unearned premium.
C. As a terminal interest.
D. As a gift of a future interest.

A

ANSWER: B. This is purely a definition question. The gift is not necessarily a taxable gift. It is a gift of a present interest. Answer C doesn’t apply to the question.

98
Q

CONCEPT: Retirement Plans
QUESTION: For the plan to be in effect for the employer’s fiscal year end, a SEP has to be established before what date?

A. Within the tax year for which the employer wishes to take the tax deduction
B. On or before the first date by which a contribution is required to be deposited
C. By April 15th of the year following the year for which the tax deduction is to be attributed
D. By the due date of the business tax return, including extensions

A

ANSWER: D. Answer A applies to qualified plans. Answer B applies to SIMPLE plans. Answer C applies to IRAs.

99
Q

CONCEPT: RBD/RMD Requirements
QUESTION: Terry is a former teacher who now works for Software, Inc. She has a personal IRA, a 403(b)-account balance from her teaching career, and now participates in a 401(k) plan at Software, Inc. Terry will turn 72 in April of the current year. She plans to continue working for Software, Inc. until she turns age 75. From which plans must she take RMDs before April 1st of next year to avoid the 50% insufficient withdrawal penalty?

A. Only the IRA
B. The IRA and 403(b)
C. All three plans
D. None of the plans shown

A

ANSWER: B. There is no indication in the question that Terry owns more than 5% of Software, Inc. She can delay distributions from the 401(k) until the year following the year she retires, regardless of her age. This exception does not apply to her IRA. The still working exception does not apply to the 403(b) because she is not currently a teacher. She falls under the IRA rules. She did not roll the 403(b) funds into her employer’s 401(k). This is no different than if she had an IRA. She would have to take distributions.

100
Q

CONCEPT: QDT
QUESTION: Marty and Libby Joseph are married and have no children. Marty is a Canadian citizen. He lives full-time in the U.S. has no wish to become an American citizen. After Libby dies, he doesn’t plan on going back to Canada. Currently Marty has about $4 million invested in real estate in Canada in his own name. Libby works as the Chief Financial Officer of a micro-cap corporation making $500,000 to $600,000 a year. Libby has completed her estate plan by establishing a QDT for Marty should she die first. If Libby dies in the current year with an estate of $17,060,000 how much should go into the trust?

A. $5,000,000
B. $12,060,000
C. $17,060,000
D. $0, because their joint exemptions protect the entire amount from federal estate tax

A

ANSWER: A. $12,060,000 will go to Marty tax free. He gets the exemption amount. The other $5,000,000 would by-pass estate taxes until he dies. It all goes into the QDT she would lose her exemption at her death. This is similar to a question in the Live Review book.

101
Q

CONCEPT: IRD (Estate)
QUESTION: Which of the following statements is correct concerning income in respect of a decedent (IRD)?

A. If the income is included in the gross estate, the estate tax attributable to that income item is generally deductible by the recipient of the income.
B. The items of income will only be subject to income tax, not estate tax. Double taxation is avoided.
C. Income that the decedent was entitled to receive but had not yet received as of the date of death is excludible in his or her gross estate.
D. An income tax refund not yet constructively received is treated as IRD.

A

ANSWER: A. Answer B is incorrect because of Answer A. An income tax refund is not IRD.

102
Q

CONCEPT: Interpolated Terminal Reserve
QUESTION: Mr. Tate died owning a whole life insurance policy under which Mrs. Tate is the named insured. The death benefit of the policy is $250,000, and the cash value was $30,000. Relative to this policy owned by Mr. Tate, what amount will be included in Mr. Tate’s gross estate for federal estate tax purposes?

A. $250,000
B. $30,000
C. The replacement value
D. The interpolated terminal reserve plus the unearned premium
E. The interpolated terminal reserve less the unearned premium

A

ANSWER: D. This is the correct way to answer the question. The amount will be greater than the cash value. Replacement value isn’t a correct answer. He died owning a policy on his wife. She was the insured; he was the owner.

103
Q

CONCEPT: Employee Benefits
QUESTION: Phillips Engineering, Inc. wants to provide an extra incentive for its key employees. They plan to hire a CFP® licensee to present a retirement planning seminar and then have the CFP® certificant prepare a full financial plan for each key employee. Regarding this

A. Phillips will have to break down the cost of the seminar and individual planning. The cost will have to be charged to the key employees because it is clearly a discriminatory benefit.
B. Only the cost of the individual financial plans must be charged to the employees; the seminar can be expensed by Phillips.
C. The program can be provided as a fringe benefit and thus be excluded from the employees’ gross incomes.
D. Because of embedded conflicts of interest, it generally violates ERISA fiduciary rules for any CFP® practitioner, who otherwise earns commissions, to provide financial planning seminars to employee groups.

A

ANSWER: A. The Tax Act 2001 Act added a fringe benefit called employer provided retirement planning services. An employee and spouse can exclude from their income the value of certain retirement planning services provided by qualified retirement plans. The exclusion will not be available to highly compensated employees unless the services are also available to each member of the group of employees. Answer A is the best choice to answer the question. The services that may be excluded are not limited to information regarding the employer’s qualified plan. It can include general advice.

104
Q

CONCEPT: Insurance
QUESTION: You are gathering data from your new client, Matt Markham, who is age 50. Matt provides you with the following information regarding his insurance coverages.
* No disability insurance
* No LTC insurance
* A PAP with $100,000/$300,000/$50,000 BI/PD liability coverage
* An umbrella liability insurance policy that requires $250,000/$500,000/$100,000 liability limits on the underlying auto policy
* Individually owned comprehensive major medical insurance having a $1,000 deductible $1,000,000 umbrella policy
What do you recommend that Matt do first?

A. Buy long-term disability income insurance.
B. Buy long-term care insurance.
C. Increase the liability limits on the PAP.
D. Decrease the deductible amount on the comprehensive major medical policy.

A

ANSWER: C. This is the simplest and fastest recommendation. Increasing the auto liability will allow the umbrella to work properly. We don’t know enough about the client to do A, B, or D. Is he/she working, retired, married, etc.? What is his/her health? The client is 50 years old. Disability insurance is subject to both health and financial underwriting. It will take time to accomplish.

105
Q

CONCEPT: RMD
QUESTION: Larry Towne turns age 72 in May. Larry is the sole owner of LT, Inc. Due to slower customer demand, he is only taking wages of $110,000 in the current year. LT, Inc. provides a 401(k) plan, but LT, Inc. only matches elective deferrals at 2% up to the first $100,000 of employee compensation. Larry is deferring the maximum amount under current year elective deferral limitations. He receives the maximum Social Security retirement benefit each year because his wages are always near or above the Social Security taxable wage base. Larry also has an IRA that he established many years ago. What do you recommend Larry do this year?

A. Roll his IRA into a Roth this year to avoid taking RMDs.
B. Stop making deferrals into the 401(k) soon because he will turn 72 and must honor RBD rules.
C. Take RMD distributions from both the IRA and 401(k) by April of next year for the current year.
D. Gather additional data to determine Larry’s cash flow needs.

A

ANSWER: B. He will be 72 this year. He must take an RMD from both his IRA and 401(k) but it is not required until next year. He is more than a 5% owner. After he takes his RMD he can roll the IRA into a Roth. Although answer D is true, this question is an RMD question. Answer C would also be acceptable.

106
Q

CONCEPT: Sharpe/Treynor/Jensen
QUESTION: Tim Brown, a new client, has engaged you a CFP® practitioner. He is asking for investment advice only. In addition to holding the CFP® certification, you are a registered representative of a FINRA broker-dealer. Tim has a portfolio of stocks he has purchased through dollar cost averaging over the years. The portfolio contains 5 stocks that are equally weighted in very different industries. In reviewing his portfolio, what composite (risk adjusted) measure of portfolio performance should you implement given his existing stocks?

A. Treynor
B. Sharpe
C. Jensen (alpha)
D. Coefficient of determination

A

ANSWER: B. Five securities are not a diversified portfolio. Some textbooks say 10-15 securities, or more are generally necessary for a portfolio to be diversified. Therefore, the portfolio has both systematic and unsystematic risk. This is a different type Treynor, Sharpe, Jensen question.

107
Q

CONCEPT: New Home Equity
QUESTION: Mr. and Mrs. Rich bought a home valued at $1,000,000 this year. The current mortgage balance is $750,000. They have decided to purchase a lot in North Carolina having a current FMV of $150,000. If the Richs’ took out a $150,000 home equity loan, the home equity loan would be subject to which of the following?

A. Passive income limitations
B. Active participation rules
C. Excess qualified residence limitations
D. A capitalization rate limitation
E. Interest expense limitations

A

ANSWER: C. New home equity indebtedness is limited to $750,000 including a home equity loan.

108
Q

CONCEPT: Early Distribution Penalty
QUESTION: Harry started taking substantially equal payments from his substantial IRA rollover account at age 55. For four years he took the required amount under the annuity distribution method which resulted in a $50,000 fixed annual payment. Then, in year five, when he was older than 59½, Harry withdrew an amount greater than the normal annuity payment of $50,000. What, if any, amount of penalty did he have to pay on the fifth-year payment?

A. 10% of $50,000 plus interest
B. 10% of the aggregate withdrawals in years 1-4 plus interest
C. 10% of the aggregate withdrawals in years 1-4 and the current year distribution of $50,000 plus interest
D. There was no penalty because Harry was 59½ when he modified the annuity distribution arrangement.

A

ANSWER: D. The penalty only applies to distributions that were made before 59½ (answer B). But the question is only asking about the year five penalty. The penalty only applies to distributions that were made before age 59½. Yes, the first 4 years would be subject to a 10% penalty, but that is not what the question is asking.

109
Q

CONCEPT: NPV
QUESTION: Which of the following is true about net present value (NPV)?

A. The investment should generally be made if the NPV is negative.
B. The investment should generally be made if the NPV is positive.
C. Its calculation discounts unequal cash flows at a required rate of return less the initial cost of an investment.
D. Its calculation can only include positive cash flow entries.

A

ANSWER: C. Answer C is the definition of NPV. NPV, while not a perfect valuation tool provides the planner with a reasonably reliable model from which to make assessments and projections. The calculation should factor both positive and negative entries.

110
Q

CONCEPT: Deferring Taxes
QUESTION: Your client, Roger, is looking for an investment that will accomplish his objective of income tax deferral. Which of the investment vehicles shown below would defer income taxes?

A. A rental apartment building
B. A municipal bond
C. A Single premium annuity
D. A Certificate of deposit

A

ANSWER: A. Investing in the rental apartment building enables depreciation deduction to offset rental income on an asset that can appreciate in value over time. The single premium annuity shown does not indicate if it is immediate or deferred. Municipal bond interest is tax exempt rather than tax deferred. Interest on Certificates of Deposit is currently taxable.

111
Q

CONCEPT: AMT
QUESTION: For purposes of calculating the AMT, which of the following is/are add-back items?

I. Private-activity municipal bond interest
II. The bargain element on exercised incentive stock options
III. Financial adviser fees
IV. NY City income tax
V. Percentage depletion from oil and gas drilling programs

A. I, V
B. II, IV, V
C. II, III, IV
D. II, IV
E. II, V

A

ANSWER: D. The bargain element of exercised ISOs, local and state income tax are “add-back” items. Answers I and V indicate preference items. Answer III is no longer deductible.

112
Q

CONCEPT: Gifting Tax Consequences
QUESTION: Kevin, a wealthy, retired individual is interested in establishing a gifting program for his 3 children and 8 grandchildren. Another financial planner is suggesting a plan that would create limited tax consequences to the donee plus provide the donee with potential future growth. Currently, Kevin owns the following assets and is considering whether to gift them.
* $1,000,000 high-yield corporate bonds
* $1,000,000 corporate zero coupon bonds
* $1,000,000 T-bills that mature in 90 days
* $1,000,000 ABC, Inc. stock with a basis of $400,000 paying $20,000 of dividends annually
What would you suggest?

A. Wait until the T-bills mature, and then give each donee $15,000 with instructions to buy a security that is similar to the ABC, Inc. stock.
B. Give each donee $15,000 worth of the zero-coupon bonds
C. Give each donee $15,000 worth of the corporate bonds
D. Give each donee $15,000 worth of ABC, Inc. stock

A

ANSWER: A. The strategy to gift cash then have the donees acquire common stock means that the donee’s basis will be the purchase price ($15,000) whereas under answer D, the donee’s basis is the donor’s(carry–over) basis ($6,000). Answers B and C have no growth capacity and could trigger kiddie tax relative to the grandchildren.

113
Q

CONCEPT: EE Bonds (GP)
QUESTION: Aaron and Linda Schwartz are married. They reported a joint AGI of $80,000 on their most recent tax return and are in the 12% marginal income tax bracket. They have one child, Stephanie, age 7. Aaron and Linda are considering purchasing “qualified” U.S. savings bonds to pay for a portion of Stephanie’s college education costs. Identify which income tax implication is true as a result of purchasing these savings bonds.

A. If Aaron and Linda purchase the bonds in their names to pay the “qualified higher education expenses” of Stephanie, the amount of their adjusted gross income at the time of redemption would be a factor in determining the tax treatment of the redemption.
B. To avoid future income tax exposure, Aaron and Linda cannot purchase “qualified” bonds in their own names for the benefit of their daughter.
C. If the bonds were purchased using an UGMA or UTMA account, income from the bonds would not be subject to the “kiddie tax” because recognition of the income can be deferred.
D. If the redemption proceeds from the EE bonds are used for “qualified higher education expenses,” no income tax is ever due.
E. When the bonds mature, Aaron and Linda can swap them for Series HH bonds and continue to defer any federal income tax exposure.

A

ANSWER: If Aaron and Linda purchase the bonds in their names to pay Stephanie’s “qualified higher education expenses” the amount of their adjusted gross income at the time of redemption would be a factor in determining the tax treatment of the redemption. If the adult owner’s AGI is below a given threshold, to the extent that redemption proceeds are used for college tuition and fees, they are free from Federal income tax exposure. Answer C is incorrect because a taxpayer may defer interest on EE bonds until the bonds mature. However, the EE bonds could be subject to the kiddie tax when redeemed. Answer D is not correct because of the word “ever”.

114
Q

CONCEPT: Basis and Exchange
QUESTION: Tommy John, who currently lives in Ohio, inherited a tract of land from his parents. Prior to dying, his parents had sold all the trees that had been on the land to the Big Axe Lumber Company. Big Axe cut all the trees, but the stumps remained. Because of this the land was only valued at $200,000 at the time of Tommy’s inheritance. It would cost Tommy $100,000 to clear the land and make is usable. Today, Mr. Swapper, land developer, approached him to discuss an exchange. Mr. Swapper now owns low lying land in Florida that has environmental issues. He cannot develop it for any commercial or residential use. While the land is on the coast, the nearest gas station is 30 miles away. After some research, Tommy believes that he can build a stilted house on Mr. Swapper’s land near the beach. Mr. Swapper keeps telling Tommy that if the land could be developed it could be worth $1 million. Presuming that Tommy takes Mr. Swapper’s offer, which of the statements below most accurately reflects the tax outcome of the exchange?

A. Because Mr. Swapper’s land is in a different state, Tommy will not be allowed to make a 1031 exchange.
B. Tommy will have a realized gain of $700,000.
C. No gain will have to be recognized by Tommy.
D. Tommy’s adjusted basis will be $800,000.

A

ANSWER: C. Under this 1031 exchange, Tommy receives no boot, therefore, his basis ($200,000 at his parents’ death) becomes his basis in the newly acquired (by exchange) Florida land.

115
Q

CONCEPT: Retirement Plan Types
QUESTION: Maxine Maxwell owns Twins, Inc. Maxine wants to install a qualified retirement plan combination of plans under which the company could maximize contributions under Section 415 but still have flexibility in regard to contributions and/or an employer matching contribution to elective deferrals. In this regard, which of the following statements is true?

A. Under ERISA rule, the plan would be disallowed because it would ultimately be discriminatory.
B. To maximize benefits to the owner, the company could install a combination of a money purchase and profit sharing 401(k) plan.
C. The company could install a defined benefit plan and a SIMPLE 401(k).
D. The company could install a profit-sharing 401(k) plan.

A

ANSWER: D. A profit- sharing 401(k) program can accomplish both of Maxine’s goals, namely flexible, and relatively large contributions. Given that up to 25% of eligible compensation may now be contributed to a profit-sharing plan, there is no longer a need for the combination of a money-purchase plan and a profit-sharing plan with 401(k) provisions. If an employer maintains a SIMPLE 401(k) plan, it cannot offer any other employer-sponsored retirement plan.

116
Q

CONCEPT: Tax Liability
QUESTION: At the local library, Todd attended a program called “You and Your 1040.” Now Todd thinks he understands how federal income tax works. He is 55, married, and earns about $150,000. Todd’s three children have all graduated from college. At work, he makes the maximum 401(k) elective deferral. He also invests in stocks that pay qualified dividends that, given his tax bracket, are taxed at a 15% rate. Todd has been trading stocks actively this year. Todd thought he figured his tax liability accurately, but his CPA said he needed to pay $10,000 more because of recent tax law changes and may get hit with a late payment penalty. You are Todd’s investment advisor and he called you for advice. How would you respond to Todd’s concern about taxes?

A. Todd, you need to educate yourself on how the recent tax law changes
B. Todd, you need to change your investment timing and allocations to avoid extra short-term gains
C. Todd, actually your taxes are relatively low. Pay the tax as your CPA advises.
D. Todd, to reduce your income tax exposure, consider diversifying your portfolio with public purpose municipal bonds.

A

ANSWER: B. Reallocating to growth stocks and reducing short-term trading should lower Todd’s federal income tax liability. Gains produced from short-term trading are subject to ordinary income tax and often, the AMT. While Answer D is reasonable, it appears that Todd prefers the equity market.

117
Q

CONCEPT: Gift Tax Techniques
QUESTION: Your client, Norman Lawson has a net worth of approximately $20 million. Norman, who is a famous litigator, is the sole owner of his highly successful law practice that now operates as a P.A. He wants to make gifts to his son and his grandchildren. Norman also owns a 30-unit residential rental building that generally enjoys profitable annual cash flow. Given the 40% gift tax rate that is likely to apply to the transfers, Norman has asked you about ways to minimize his potential gift tax liability. After you explain discount valuation to Norman which of the following techniques might be available to Norman?

A. Transfer the rental building to a newly established family limited partnership then gift limited partnership interests to family members
B. Make an installment sale of stock in the law practice to family members
C. Gift stock in the law practice to family members
D. Arrange a gift-leaseback relative to the rental property

A

ANSWER: A. The interests transferred to family members will be limited partner interests. Thus, they should qualify for both lack of marketability and lack of control (minority) discounts as they are valuated for federal gift tax purposes. Norman will not benefit from gifting P.A. stock to non-professionals. Answer D does not accomplish Norman’s objective because it is generally used to transfer income, rather than assets alone, to other family members. Further, the gift leaseback technique offers no discounting.

118
Q

CONCEPT: MEC
QUESTION: Mr. Dorsey, age 54, purchased a universal life insurance contract in 2003 when he was much younger. He named his wife as its beneficiary. She is living. The stated annual premium was $2,000. For each of the first 6 years he paid the $2,000 premium. However, at the end of the 7th year, he inherited $300,000 from his great Aunt Minerva and, he paid $38,000 to the insurance company. He did not want to pay further premiums in the future. The contract’s cash value is currently $80,000. Regarding Mr. Dorsey’s situation, which of the following statements is (are) true?

I. If Mr. Dorsey dies, the death benefits are income tax free to the beneficiary named under the policy. Mrs. Dorsey is living at the time of Mr. Dorsey’s death.
II. If Mr. Dorsey withdraws (rather than borrows) $50,000 from the policy, the withdrawal is tax-free.
III. If Mr. Dorsey surrenders the contract, the $40,000 increase over basis is taxed as long-term capital gain.
IV. If Mr. Dorsey borrows $40,000 from the policy, only $30,000 is subject to ordinary income tax plus a 10% penalty.
V. If Mr. Dorsey borrows $38,000 from the policy, the $38,000 is subject to ordinary income tax rates plus a 10% penalty.

A. I, II, III, IV
B. I, II
C. I, III
D. I, IV
E. I, V

A

ANSWER: D. When Mr. Dorsey paid the lump sum, the policy became a MEC. His basis is $50,000, the cash value amount is now $80,000. ($80,000 - $50,000 = $30,000) The $30,000 will be taxed as ordinary income plus a 10% penalty. Non death distributions, including withdrawals, loans, or surrender, will come out under LIFO rules.

119
Q

CONCEPT: Gifted Business Interests
QUESTION: Mac Blair decided to make a gift of Blair, Inc. common stock to his son, Blake. Mac seeks that any future appreciation of the stock is not included in his estate for federal estate tax purposes. He is going to retire soon and will need income during his post-retirement years. Mac has converted the majority of his common stock to preferred stock and gifted the remaining common stock to his son. What is the result?

A. The value of the common stock for gift tax purposes will be based on dividends paid on the preferred stock.
B. The value of the preferred stock for gift tax purposes will be based on the FMV of the business less the value of the common stock.
C. The value of the common stock for gift tax purposes will be based on the total FMV of the business.
D. The value of the common stock for federal gift tax purposes will be based on a business appraisal performed by a qualified appraiser.

A

ANSWER: A. The preferred share value will be determined based on the stated dividends. The common stock value would then be the difference between the FMV of the corporation and the aggregate value of the preferred shares

120
Q

CONCEPT: Social Security
QUESTION: John, age 69, and Mary, age 69, (married) both worked until this year. Throughout the last 35 years, both had high paying jobs that consistently exceeded the maximum Social Security taxable wage base. As a result, both his and her benefits will be in the same amount when they claim them at age 70. They will each receive over $3,000 per month. If John predeceases Mary, what will happen to Mary’s benefits?

A. The amount will not change. Mary will still receive her benefits.
B. Mary will receive her benefit plus ½ of John’s
C. Mary will receive her benefits plus John’s benefit
D. Mary’s benefit will reduce (FRA benefit) because John predeceased her.

A

ANSWER: A. After John’s death, Mary will be entitled to the greater of her benefits or 100% of John’s benefits. John and Mary had matching amounts at or over $3,000 per month. That amount will continue.

121
Q

CONCEPT: Financial Planning Fundamentals
QUESTION: Alice and Leo Fuller (both age 30) have come to you. Alice earns $60,000/year working full time as an employment counselor. Leo earns $50,000/year working full time as a retail sales manager. Now Allice wants to have children and then work only part/half-time. Fullers have accumulated $300,000 in savings in short-term fixed income investments. Their three-bedroom home has a current FMV of $225,000. The current mortgage balance is a $150,000 and the note carries a 4% fixed interest rate. The Fullers want to retire at age 55. Given their objectives, how should the Fullers proceed?

A. They should sell the home and rent. If Alice works only works part-time and they have children, the Fullers cannot afford the house.
B. They should keep the home. Even if Alice works only part-time, the Fullers can still afford to keep the home and have room for children.
C. The Fullers should use some of their savings to pay down the mortgage to a level where the PITI can be covered under their reduced income.
D. Given their objectives, renting will give the Fullers the flexibility they need

A

ANSWER: B. Presuming that the Fullers have children, they may need a bigger house. The home usually an effective inflation hedge. One generally can’t pay down a mortgage to reduce PITI. PITI remains relatively constant even as the mortgage is paid down. The loan will be paid off faster, but the mortgage agreement (PITI) would remain relatively unchanged. The $225,000 home does not seem extravagant.

122
Q

CONCEPT: Bond features
QUESTION: Mrs. Pratt, who has always been a conservative investor, expects interest rates to decline. She wants to purchase some high yield debt to take advantage of current interest rates. Which of the following bond features would best help Mrs. Pratt relax about potential interest rate changes going forward?

A. A put feature
B. Call protection for 10 years
C. A premium bond
D. A conversion feature
E. A discount bond

A

ANSWER: B. Ten years of call protection enable Mrs. Pratt to enjoy the relatively high rates for at least ten years. Bond premiums and discounts are not features. Given current “high rates,” Mrs. Pratt would not be anxious about rates rising soon. A put feature would matter if she feels interest rates will rise. Further, both the put feature and conversion feature would reduce yields.

123
Q

CONCEPT: Educational Funding
QUESTION: Mrs. Sikes purchased a qualified state tuition program for each of her two granddaughters. If she paid $80,000 into each account in 2022, which of the following is true?

A. She will not ever be able to contribute to a Coverdell ESA for either granddaughter.
B. She will be able to contribute to a Coverdell ESA for either granddaughter this year using her lifetime applicable gift exemption amount.
C. She will not be able to contribute to a UTMA account this year.
D. $64,000 of each $80,000 transfer is a taxable gift.

A

ANSWER: B. The Coverdell contributions are not affected by the contribution to the qualified state tuition program. Answer A is wrong because of the word “ever”. Mrs. Sikes can contribute to a UTMA account, but she would have to use her applicable gift exemption amount ($12,060,000). It is available after the $16,000 annual exclusions are used up. Answer D is not true. Presuming that Mrs. Sikes lives for five full years following the transfer, she has not made a taxable gift. If she dies in the five-year post contribution period, part of the $80,000 transfer would be thrown back into Mrs. Sike’s gross estate.

124
Q

CONCEPT: Investment Choices
QUESTION: Wesley graduated from MIT 5 years ago. His master’s thesis addressed super conductor wave analysis. He works for a small specialty company that does not provide a menu of employee benefits. Other than health insurance, the company offers no other benefits. Wesley’s base pay is $10,000 per month. He usually receives a 15% bonus at year end. He lives in a small, well-appointed apartment and walks to work. He is saving about 50% of his gross income. He has accumulated about $200,000. To date, Wesley has accumulated in a money market deposit account at his local bank. He is asking you for effective recommendation relative to his assets and cash flow. From the recommendations below, which would you suggest to Wesley?

A. Wesley should establish an emergency fund of at $80,000 and invest the remainder in AAA rated municipal bonds.
B. Wesley should buy a condominium making a 20% down payment on a condominium having a FMV of $200,000 condominium. He should be able to deduct the mortgage interest and property tax.
C. Wesley should buy a $1,000,000 variable life insurance policy using a single premium and keep about $40,000 in the money market deposit account.
D. Wesley should contribute to a deductible IRA each year and invest $4,000 to $6,000 a month in diversified portfolio of ETFs.
E. He should contribute to a Roth IRA each year and invest monthly in high tech companies with strong commitments to development.

A

ANSWER: D. Wesley’s tax rate is 24% and he will likely make too much to fund a Roth IRA. The emergency fund indicated in Answer A seems larger than it needs to be given Wesley’s age and income. The risk/return on the municipal securities seems too conservative. Without mention of family members the need for life insurance is unclear. Nothing indicated that Wesley has high investment risk tolerance: The focus on tech stocks is aggressive and poorly diversified. He is not in a plan at work. Therefore, he can do a deductible IRA.

125
Q

CONCEPT: Financial Authorities (GP)
QUESTION: Sam believes he was cheated on an investment transaction with broker/dealer with which you are not affiliated. As a financial planner, if Sam is seeking recourse, how would you recommend that he proceed first?

A. Sam should notify the CFP Board
B. Tell Sam to hire an attorney and sue the broker/dealer that handled the transaction.
C. Tell Sam to report the transaction to the FINRA and SEC
D. Take the broker/dealer to arbitration through the FINRA procedures
E. Write to the compliance officer (registered principal) of the broker/dealer that handled the transaction and ask for his money back

A

ANSWER: E. A disgruntled client of a broker/dealer should generally begin the complaint process through that broker/dealer’s compliance function. If the firm is unresponsive or disagrees with the customer’s claim, arbitration should be pursued through FINRA. Mediation may also be available. The broker/dealer community is governed by FINRA directly under the supervision of the SEC. The CFP Board is not the regulator for broker/dealers.

126
Q

CONCEPT: Licenses (GP)
QUESTION: An individual holding a FINRA Series 6 license can receive commissions relative to the sales of which of the following investments?

I. Mutual funds
II. Variable life insurance
III. Variable annuities
IV. ETFs

A. I, II, III, IV
B. I, II, III
C. I, IV
D. II, III
E. I only

A

ANSWER: E. A Series 6 limited securities representative licensed individual can sell mutual funds only. To sell variable products, the individual must also hold a state-issued insurance producer’s license. The question does not indicate any insurance license. ETFs are Exchange traded funds. They are normally traded on a national exchange without a prospectus. To earn commissions for sales of products not accompanied by a prospectus, one must hold the FINRA Series 7 general securities representative’s license.

127
Q

CONCEPT: Gift basis
QUESTION: Sally gifted property with a basis of $55,000 to her daughter, Alice. The FMV on the date of gift was $110,000. The gift was subject to gift tax at a 40% rate. What is Alice’s adjusted basis in the property for determining her gain or loss when she sells the property?

A. Alice’s basis will be increased by the gift tax paid by her mother that is attributable to the appreciation.
B. Alice’s basis in the property is its FMV as of the date of the completion of the gift.
C. Alice’s (carryover) basis is $55,000.
D. Alice’s basis does not matter because she will realize neither gain nor loss when she sells the property.

A

ANSWER: A. Basis to a done is increased by the gift tax paid by the donor that is attributable to the appreciation of the gift.

128
Q

CONCEPT: Contribution Limits
QUESTION: Susan works as a part-time employee for the local school system. She earns approximately $10,000 annually. Susan also has a small eyebrow shaping business that nets her $30,000 in a typical year. Can she participate in both a TDA/403(b) through the school system and a profit-sharing plan through her own business?

A. Yes, Susan can participate in both plans because the employers are unrelated.
B. No, Susan can only participate in the profit-sharing plan because she does not meet the 1,000-hour rule under ERISA.
C. No, Susan can only participate in the TDA because self-employed persons cannot provide profit-sharing plans.
D. No, Susan may participate in either plan but not both.

A

ANSWER: A. Susan may participate in both plans. Employee status includes part time employees. There is no 1,000 ERISA requirement for participation in a 403(b) program. Susan can install a profit-sharing Keogh plan if her business is unincorporated and a traditional profit-sharing plan if her business is a corporation.

129
Q

CONCEPT: Retirement Plan Types
QUESTION: Sonia Appletree owns an upscale retail gift shop. Business varies so much that she cannot hire full-time employees or provide benefits. She has decided to limit the number of full-time workers to two employees. The remainder of her employee needs can be met by prior employees who only want to work part-time, typically 400 hours per year at $10 per hour. What kind of benefits would you suggest to Sonia that would cover herself and the full-time employees and would exclude the part-time employees?

I. A profit-sharing plan
II. A SEP
III. A group health insurance plan
IV. A SIMPLE 401(k)

A. I, II
B. I, III
C. I, IV
D. II, III
E. III, IV

A

ANSWER: B. Under ERISA rules, a profit-sharing plan may exclude from participation employees working fewer than 1,000 hours per year. Eligibility to participate in an employer-provided group health insurance plan normally requires 32 hours per week. SEP eligibility falls under the 3-year rule which could force Sonia to cover certain returning part time workers, The SIMPLE (401k) implies employee deferrals. At this point it is not clear as to whether the employees would want to make elective deferrals. Her own maximum contribution would be lower than that under profit- sharing plan.

130
Q

CONCEPT: DDM Formula
QUESTION: Consider the information below regarding Stocks A and B:
* The risk-free rate of return is 5%.
* The market return is 10%.
* Stock A is currently selling for $19 with a beta of 1.2. Dividends are currently $.90 and are expected to increase by 5% per year.
* Stock B is currently selling for $60 with a beta of .7. Dividends are currently $1.25 and are expected to increase by 6% per year.
Using the DDM formula which statements below are true?
V = D1 (divided by) and r = rf+ (rm - rf)
r-g
NOTE: The formulas would be given on the formula sheet.
I. Stock A is overvalued.
II. Stock A is undervalued.
III. Stock B is overvalued.
IV. Stock B is undervalued.

A. I, III
B. I, IV
C. II, III
D. II, IV

A

ANSWER: A. Both stocks are overvalued.
Stock A r = 5% + (10%-5%) 1.2 = 5% + (5%) 1.2 = 11%
A’s Price = $.90 (1 +.05) = 15.75
.11-.05
Stock B r = 5% + (10% - 5%) .7 = 8.5%
B’s Price = $1.25 (1 + .06) = $53.00
.085 - .06

131
Q

CONCEPT: Qualified Withdrawals
QUESTION: Mr. Adams, age 52, just received an excellent job offer and is going to leave his current employer. He has $300,000 in his account in the employer’s qualified plan (fully vested). Mr. Adams isn’t sure what to do with the $300,000 now. However, he feels he may need some of the money in 4 years. He plans to work for a competitor company for the next 4 years. His new employer provides a 401(k) plan. What do you recommend he do?

A. Roll the $300,000 qualified account balance into an IRA
B. Take substantially equal payments until age 60 and then stop the payments
C. Roll the $300,000 into his new employer’s 401(k)
D. Take a cash distribution of his current account balance from the current employer’s qualified plan.

A

ANSWER: C. There will be no adverse tax consequences if Mr. Adams rolls the current account balance into his new employer’s 401(k) program. A qualified plan, like a 401(k), triggers no 10% penalty for separation from service after age 55. The age 55 exception does not apply to IRAs. Mr. Adams is now age 52; in four years he will be 56 not 59½, so the premature withdrawal penalty would apply unless he arranges substantially equal payments. In four years, Mr. Adams may need more money than that. He can also take out a loan from the 401(k).

132
Q

CONCEPT: Company Bankruptcy
QUESTION: What is the order of liquidation payout if a corporation goes bankrupt?

I. Mortgage bondholders
II. Common stockholders
III. Debenture bondholders
IV. Secured creditors
V. Preferred stockholders

A. I, II, III, IV, V
B. I, IV, III, V, II
C. III, II, V, IV, I
D. IV, V, II, III, I

A

ANSWER: B. Secured creditors such as mortgage bondholders and equipment trust certificate holders are senior. Second are unpaid wages, taxes, and trade creditors. Third are debenture bondholders followed by preferred shareholders. Common shareholders are the most junior and therefore last.

133
Q

CONCEPT: Financial Planning Process
QUESTION: Mrs. Andrews is referred to you by a current client who happens to be her employer. The referring client is a physician and Mrs. Andrews works as the Medicare billing administrator in his office. She is divorced, and her income barely covers her living expenses. She wants to talk with you about her daughter. She asked you to meet with her daughter and son-in-law. You agree.

Her daughter, Leslie, and her husband Tommy meet with you. Both are employed in hourly wage jobs and have no savings. Fortunately, their employers provide them with group health insurance. Leslie is pregnant and the expected due date is 2 months away. That is all the financial information they can provide. What you recommend that Leslie and Tommy do?

A. Establish an emergency fund to cover 6 months of fixed and variable expenses.
B. Buy term life insurance on Tommy’s life with a death benefit of at least $500,000.
C. Suggest that Mrs. Andrews tighten her belt so she can help them with money.
D. Help Leslie and Tommy determine whether they qualify for any local or state financial assistance, like food stamps.

A

ANSWER: B. This growing family will need life insurance. This is especially true because Leslie and Tommy can’t rely on Mrs. Andrews for financial help. With term coverage they should be able to obtain a reasonably high death benefit. With two months before the baby comes and limited wages a six-month emergency fund does not seem reasonable. Further, they will have extra cash needs for baby-related expenses. Answer C is incorrect because Mrs. Andrews is not the client for whom the recommendation in the question is to be made.

134
Q

CONCEPT: Retirement Plan (Retirement)
QUESTION: Alex Hamilton, now age 57, quit his job 2 years ago. After being married for 30 years. Alex and his wife decided to split when he quit his middle management job and started developing websites for the handicapped. His now ex-wife took half of their money and moved to France. Alex’s websites went viral and his annual income went from around $90,000 in a typical year to $300,000. He has no formal employees but some of his fraternity brothers help him and are paid under a 1099 arrangement. Alex wants to establish a retirement plan. You ask him about his lifestyle and his expenses. He says he is now dating the lovely Fifi, who is age 39. To compete with her other boyfriends, he leased a Mercedes-Benz and takes Fifi to fine restaurants and the theater where he buys main floor seats. From the choices below, which type of retirement plan would you suggest?

A. A SEP
B. A Uni-(k)
C. A defined benefit Keogh
D. An age-weighted target benefit Keogh

A

ANSWER: B. The catch-up provision available with the Uni (Solo) 401(k) allows Alex to contribute a total of $67,000 for the current tax year. Given that they do not offer a catch-up contribution opportunity, the SEP and target benefit only allow $61,000. The Uni-(k) will be an ERISA plan. Fully funding actuarially determined contributions to a defined benefit plan will require more money than his current expenses allow.

135
Q

CONCEPT: DDM
QUESTION: Arthur is looking to buy RST common stock for $100. The current dividend is $2. The stock is expected to grow its dividend by 6% for three years, and then by 8% thereafter. Arthur’s required rate of return on this stock is 10%. Should he buy the stock?
V = D1 / (r-g)

A. No, based on the DDM, the stock appears to be overvalued.
B. Yes, based on the DDM, the stock appears to be undervalued.
C. Yes, based on the DDM, the stock will satisfy Arthur’s required rate of return.
D. Yes, because the dividends would only be taxed at 15%.

A

ANSWER: B. By factoring the DDM, we know that the current market price of $100 is less than $108.00.

V = $2 (1 + .08) = $2.16 = $108.00
.10 - .08 .02

In the formula above, the 2nd growth rate is used to calculate the intrinsic value with an adjustment upward or downward relative to whether the early years’ dividend is lower or higher than the later years’ dividend

136
Q

CONCEPT: Preferred Stock
QUESTION: What is an advantages of buying preferred stock?

A. Warrants can be attached, adding long-term value to the preferred stock.
B. Rights can be attached, adding short-term value to the preferred stock.
C. 50% of the dividends received by a domestic corporation are tax exempt.
D. Both A and C

A

ANSWER: Both statements A and C are correct. A subscription warrant may be issued with a bond or preferred stock offering. A warrant is usually issued as a sweetener to make the issue more attractive. It entitles the holder to buy a proportionate amount of the issuer’s common stock at a specific price. Presuming the corporation buying the preferred shares has no ownership in the distributing corporation, seventy percent of the dividends received are tax exempt. Preemptive rights are available to common, rather than to preferred shareholders. If the corporation wishes to issue more shares, common stockholders have the right to buy these shares before they are offered to the public.

137
Q

CONCEPT: ISOs
QUESTION: Mr. Baldwin receives 10,000 ISOs from his employer to purchase LMN Corporation stock at $10 per share. Within two years of the grant date, he exercises them at $25 per share. Several years later, Mr. Baldwin sells the 10,000 shares of LMN for $100 per share. Regarding federal income tax as it applies to this situation, which of the following statements are true?

I. There is no taxable event on the grant of the options.
II. Mr. Baldwin will realize $150,000 of additional income for tax purposes upon exercise.
III. Mr. Baldwin will realize a long-term capital gain of $900,000 when he sells the stock.
IV. Mr. Baldwin will realize a long-term capital gain of $750,000 when he sells the stock.

A. I, II, III
B. I, II, IV
C. I, III
D. I, IV

A

ANSWER: A. The two required holding periods to create long-term gain relative to an ISO reflects 1) grant to sale (2 years); 2) exercise to sale (1 year). Before selling the exercised ISO shares, Mr. Baldwin held them at least one year from the date of exercise and at least two years from the grant date. This is a qualified disposition. Mr. Baldwin exercised the ISOs at $10 per share creating a basis of $100,000. Nothing in the question indicates that the bargain element (an AMT preference item) actually triggered AMT to Mr. Baldwin. The long-term capital gain at the time of the sale is $90 per share.

138
Q

CONCEPT: Medicare Part A
QUESTION: Which of the following benefits would be covered under Medicare Part A?

I. Home health services that are part of a treatment plan
II. First 3 units of blood in a calendar year
III. Oral anti-cancer drugs
IV. Care in an extended care facility (maximum 100 days)
V. Emergency care while in Europe

A. I, II, V
B. I, IV
C. II, IV
D. III, V

A

ANSWER: Medicare Part A will cover home health visits that are part of a treatment plan and up to 100 days in an extended care facility presuming rehabilitative purposes. The patient must pay the hospital cost for the first 3 units of blood in a calendar year or have blood donated by the person or someone else. Answer III is covered by Part B. Answer V is not covered by Medicare.

139
Q

CONCEPT: American Opportunity Credit
QUESTION: The American Opportunity credit reflects which of the following?

A. 100% of the first $1,200 of qualified expenses and 50% of the next $1,200 of qualified expenses incurred during college
B. $2,000 maximum
C. Applied to the first $2,500 of qualified expenses incurred for the first two years of college
D. 100% of the first $2K of qualified expenses and 25% of the next $2K qualified expenses incurred during college

A

ANSWER: D. The American Opportunity Credit is limited to 100% of the first qualified expenses up to $2,000 and 25% of the next $2,000 of qualified expenses incurred for post-secondary education. Answer A is wrong because it is referring to Hope credit numbers. The Hope Credit is no longer available. Answer B is wrong because the AOC is currently a maximum of $2,500 maximum. Answer C is wrong because the AOC applies to the first $4,000 of expenses during all 4 years of undergraduate school.

140
Q

CONCEPT: Ethics
QUESTION: For a while you have sensed that Edwin Edwards is no longer mentally competent. While monitoring his investment activities, you learn that he is considering giving his power of attorney to an individual with a history of elder abuse. Whom should you call?

A. The compliance officer (registered principal) at your broker-dealer
B. Edwin’s attorney
C. Edwin’s children
D. His investment advisor

A

ANSWER: B. Edwin’s attorney enjoys “client privilege” and cannot be forced to discuss Edwin’s condition with third parties. The question does not indicate that his investments are with your broker-dealer. Edwin’s children are not your clients. You do not want to violate Confidentiality in that way. Discussing Edwin’s condition with the investment advisor also breaches Confidentiality. You could also speak with Edwin’s physician but that is not among the answer choices.

141
Q

CONCEPT: NPV
QUESTION: Your client Barbara Barrington owns a business worth $2,000,000. If she requires a 12% return based on a recent offer to buy the business, what is Barbara’s business actually worth?
Offer 1
$400,000
$450,000
$550,000
$600,000
$700,000

A. $1,795,131
B. $1,885,869
C. $2,112,173
D. - $114,131

A

ANSWER: B. Because we do not know what Barbara initially paid to acquire the business, use a zero in the first year to solve for the present value.
HP12C
0 g CFo
400,000 g CFj
450,000 g CFj
550,000 g CFj
600,000 g CFj
700,000 g CFj
12 I
f NPV

142
Q

CONCEPT: TRUST TYPE
QUESTION: Lenny was divorced. He has two daughters with his first wife. A few years before his death, he married Marilyn (second wife). Lennie established a trust for Marilyn. The trust provisions gave Marilyn the right to trust income limited to the ascertainable standards of health, education, maintenance, and support (HEMS). The trust agreement also provides Marilyn with a discretionary right to principal, limited to the same HEMS standard, but which had to be preceded by the exhaustion of Marilyn’s other resources. After Marilyn’s death, the remainder of the trust passes to Lenny’s children. What type of trust does it appear that Lenny established?

A. A qualified terminable interest property (QTIP) trust
B. A qualified domestic trust (QDT)
C. A Bypass trust
D. A power of appointment trust
E. An estate trust

A

ANSWER: B. The provisions for Marilyn reflect a bypass trust (B trust). The right to income limited to HEMS is not a right to all income from the trust as would apply with a QTIP trust. Also, the QTIP cannot use the lifetime exclusion.

143
Q

CONCEPT: Ethics
QUESTION: According to the CFP Code of Ethics, a certificant shall disclose to a prospective client or clients the following information in an accurate and understandable description of the compensation arrangements being offered which must include:

I. Information related to costs and compensation to the certificant and/or the certificant’s employer.
II. Terms under which the certificant and/or the certificant’s employer may receive any other sources of compensation, and if so, what the sources of these payments are and on what they are based.

A. I only
B. II only
C. Both I and II
D. Neither I nor II

A

ANSWER: C. Both Statements I and II are correct. CFP® certificants are required to disclose compensation to prospective and current clients.

144
Q

CONCEPT: Life Insurance Provisions
QUESTION: Mr. Pate, a widower, is age 65. He is in poor health due to emphysema and a heart condition. Unless his neighbors pick up Mr. Pate’s mail at the post office, it just piles up. He is concerned about the premiums for his life insurance policy. Which of these life insurance contract provisions will help keep the policy in force?

I. Renewability
II. APL
III. Reinstatement
IV. Disability waiver of premium
V. Grace period

A. I, III, IV
B. I, V
C. II, III, V
D. II, V
E. III, IV, V

A

ANSWER: D. APL is the abbreviation for the automatic premium loan. This provision will pay the premium if it remains unpaid beyond the grace period. If Mr. Pate had claimed disability waiver, there would be no premium due notice. A disability waiver of premium provision generally means that the insurer will then pay the premium for the life of the contract with no future premium notices. Exercising the reinstatement provision would mean that Mr. Pate he would have to provide proof of insurability (medical information) in order to reinstate the policy if it lapses. Due to Mr. Pate’s health reinstatement will probably not be available.

145
Q

CONCEPT: Bond Risk
QUESTION: When seeking to educate a client, what should you stress as the greatest risk associated with investing in bonds?

A. Market interest rates
B. Price volatility
C. Call protection
D. Bond rating

A

ANSWER: B. While many investors erroneously presume that investing in bonds does not carry risk to principal, that is not so. Bond prices fall with rising interest rates. If an investor cannot hold the bond to maturity and thus sells it when rates are higher relative to the coupon rate of the bond, that bondholder is likely to lose principal. If interest rates have risen sharply the loss of principal can be significant.

146
Q

CONCEPT: Tax Deductions
QUESTION: This year Joan began helping her mother, Emma, with cash payments (totaling $10,000) this year. Emma lives on a limited fixed income plus Social Security benefits ($6,000 yearly). May Joan claim any tax deductions for the direct cash payments?

A. No, the cash payments are considered to be gifts.
B. No, only gifts to a public or private charity are tax deductible.
C. Yes, Joan can claim the dependency exemption for Emma if her mother’s limited fixed income is gross taxable income and less than $4,300 yearly (2021)
D. Yes, Joan may claim the dependency exemption for Emma as long as she provides more than half of her mother’s support

A

ANSWER: A. The money is a gift. Personal exemptions have been eliminated after 2017.

147
Q

CONCEPT: Foreign Person Estate
QUESTION: The late Ronald Carlisle, a Canadian citizen was married to Judith, a U. S citizen. He was wealthy and owns a substantial portfolio of stocks which trade in both Canadian and American markets. He also owned precious metals and substantial real property (resorts) in both Canada and in Michigan. When Ronald died, can the U.S. impose estate tax on Ronald’s estate? Why or why not?

A. Because Ronald was not a U.S. citizen the U.S. has no right whatsoever to impose federal estate tax on his estate.
B. Although Ronald was not a U.S. citizen because he was married to a U.S. citizen (Judith) the U.S. may levy federal estate tax on his entire estate.
C. If Ronald had established a qualified domestic trust (QDOT) all U. S. federal estate tax could have been avoided.
D. Because Ronald owned real property in the U.S. as well as U.S. stocks U.S. federal estate tax may be levied relative to those specific properties.

A

ANSWER: D. Although Mr. Carlisle is not a U. S. citizen, any real property he owns that is situated in the U.S., as well as any U.S. stocks he owns may be subject to federal estate tax in the US. A QDOT is appropriate when the transferor is a U.S. citizen. Ronald is not a U.S. citizen.

148
Q

CONCEPT: Expensing/Capitalizing/Deducting
QUESTION: In order to sell her property, your client, Diana Draper must have the property graded level, seeded, and surveyed. The total cost for these services is $20,000. Which of the following can you accurately tell Diana about the $20,000?

A. The $20,000 can be expensed.
B. The $20,000 must be capitalized.
C. The $20,000 can be expensed by making a Section 179 election.
D. The $20,000 can be expensed by making a Section 197 (amortization) election.

A

ANSWER: B. Improvements that add to the value of land (in this case by making it more marketable) or adapt it to new uses are capital improvements. Capital improvements generally increase basis.

149
Q

CONCEPT: MEC
QUESTION: Adam Brooks, who was age 60 and married, died yesterday. He owned a $500,000 universal life insurance policy with a death benefit that increases along with growth in the cash savings account (Option B). Adam bought the policy 6 years ago by making a $50,000 single premium payment. The carrier at the time of the purchase had informed Adam that the policy was a MEC. At the time of Adam’s death, the cash value had grown to $70,000. His wife Jane, age 59, is the primary beneficiary. Jane has asked you how much of the death proceeds will be included in Adam’s gross estate. How would you accurately respond to Jane?

A. Nothing would be included in Adam’s estate because the policy passes by the marital deduction to Jane free from federal estate tax.
B. $20,000 ($70,000-$50,000) would be in Adam’s gross estate plus a 10% penalty because the policy is a MEC and Jane, the beneficiary is under 59½.
C. $550,000
D. $570,000

A

ANSWER: D. Under the increasing B option for universal life insurance, the amount in the cash savings account is added to the base death benefit to, in this example produce a total benefit of $570,000. Because Adam owned the policy at the time of his death that amount would be included in his estate for federal estate tax purposes. MEC rules only affect distribution other than death benefits.

150
Q

CONCEPT: Form 709, Gift Splitting
QUESTION: John and Pamela Underton wrote a check for $500,000 to their daughter, Bunnie, on January 1st of the current year. It was a gift. They wrote the check out of their joint money market account. John called his CPA to inform him that the gift had been made and requested that the CPA prepare two Form 709’s to report the gift splitting. Before the 709 was filed, John died. Bunnie had not cashed the check as of the date of her father’s death. Given these events, what tax results would be expected?

A. The amount of the check would be considered a gift of a future interest.
B. Because Bunnie had not cashed the check as of the date of her father’s death the transfer will be considered an incomplete gift.
C. The gift will be presumed to have been made by Pamela alone.
D. The transfer will still be presumed as a split gift.

A

ANSWER: D. Gifts made before one spouse dies may be split even if that spouse dies before signing the appropriate consent and election on Form 709. On behalf of the deceased spouse, his executor can make the appropriate election. This is not per se an incomplete gift. The check was written from a joint checking account and Pamela (the mother) is still living.

151
Q

CONCEPT: Financial Planning Prioritization
QUESTION: You are advising top management of the Royal Ribbon Corporation. Royal Ribbon recently hired James White, a promising information technology executive. James fits the company need and Royal Ribbon wants to retain him. However, following a background check the company learned that James had filed for Chapter 7 bankruptcy and lost his house to foreclosure. Royal Ribbon Corporation has engaged you to counsel James regarding his financial affairs. In your first meeting with James, you learn that he is married with 3 small children. His wife is currently staying at home, but she hopes to return to work as a systems analyst as soon as they can afford day care. They are renting a two-bedroom apartment. There are no investments. The money in the White’s checking account is enough to cover monthly expenses. You are reviewing the company benefits with James. The health insurance program offered to employees through Royal Ribbon, Inc. is a PPO plan. The cost of family coverage is $900 per month. The group disability insurance program provides total disability benefits under a split definition that reflects a 5-year own occupation, then modified any occupation to age 65 based on a $5,000 per month maximum benefit. The plan is 40% contributory (by James). Royal Ribbon, Inc. provides employees with the opportunity for a 6% maximum deferral into their 401(k) program which also offers a 3% match. How would you suggest that James prioritize his financial activities?

I. Enroll in the PPO plan.
II. Enroll in the group disability income insurance plan.
III. Participate in the 401(k) plan.
IV. Establish a 6-month emergency fund.

A. I, II, III, IV
B. I, II, IV, III
C. II, IV, I, IIII
D. IV, III, I, II

A

ANSWER: A. With a wife and 3 small children above all, James needs medical insurance. The last priority is contributing to the 401(k). Disability insurance is the next highest priority because as of the moment, James is the only income earner. Yes, the emergency fund is also important.

152
Q

CONCEPT: Rollover Rules
QUESTION: Samuel Hall, age 69, is about to retire. He participates in his not-for-profit employer’s 403(b) plan. However, Samuel would like to avoid taking RMDs as he has cash set aside for retirement. He would like the proceeds of the 403(b) to pass to his children. Which of the following strategies would be available to Samuel?

A. At retirement, roll the 403(b) into a Roth IRA
B. At retirement, take RMDs with his children as primary beneficiaries
C. At retirement, roll the 403(b) into an IRA and then into a Roth IRA
D. At retirement, take distributions

A

ANSWER: A. Under current rules, distributions from tax-qualified retirement plans, TSAs [403(b)] and governmental 457s may be rolled directly into a Roth IRA. Samuel will have to pay tax on the amount that is rolled over. There would be no penalty because rollovers do not incur penalty and because Samuel is well over age 59½.

153
Q

CONCEPT: Retirement Plan Type
QUESTION: Advance Tech, a closely held S corporation, wants to provide an employee benefit that will help it to retain employees (100+). The company is profitable but typically has no extra cash. The owners are comfortable to permit employees to share in the company’s growth. What employee benefit would you recommend?

A. A profit-sharing/401(k) plan
B. A non-contributory stock purchase plan (ESOP)
C. A Simple plan
D. A SEP plan
E. A Money-purchase pension plan

A

ANSWER: Advance Tech does not have the means to make cash contributions. But the corporation could contribute stock. The owners are comfortable to have employees share in the growth of the company. The profit-sharing plan would require a substantial and recurring (although not annual) cash contribution. The 401(k) provision would generally entail employer matching contributions of cash. NOTE: The 100+ employee population eliminates the SIMPLE.

154
Q

CONCEPT: Financial Planning Process
QUESTION: Your client Clara Bow, age 80, has enjoyed a full life. Oscar, her husband of 60 years recently died. Because he had several serious health issues, his death it was not unexpected. During the last years of Oscar’s life, they traveled extensively and stayed at 5-star hotels. Oscar was an inventor. His patent royalties produce more income than she can spend. Clara’s charitable wishes focus on the church she has attended throughout her lifetime. She was both baptized and married there. Clara’s family includes two married children plus 4 sets of married grandchildren. Oscar did extensive investing with you and after he died you continued to advise Clara regarding investments and financial matters generally. Before he died, Oscar told you that many of the family members relied on gifts and loans from their parents. Oscar had been frustrated that his children and grandchildren had not been more self-reliant. Now Clara wants advice from you as to gifting to family members. What would you recommend to Clara?

A. Limited gifting to the excess money she is not spending.
B. Clearly understand the annual gift tax exclusion limit, the overall limit on tax-free lifetime giving and how the Form 709 works.
C. Gather more data including a current and detailed Statement of Cash Flows. Design and implement uniform gifting allocation to all family members.
D. Explain to her the $12,060,000 gift tax exemption and how it is factored into transfer tax on Form 706.

A

ANSWER: C. A gifting strategy should be proceeded by careful cash flow analysis. Clara may have health problems, where the excess cash flow would be needed. There is no indication of Clara’s net worth or how long the patent rights will continue. Given her age, Clara may need some of her income for health related and other reasons.

155
Q

CONCEPT: Section 303, Section 6166, Section 2032A
QUESTION: Lawrence Letterman died with a gross estate of $20 million. His adjustments to his gross estate are $1,000,000. He was single and his estate bequeathed all his assets to his daughter, Lulu. Lawrence’s major asset was a cattle ranch that operates as a corporation. As of the date of Lawrence’s death, the corporation was worth $15 million, and the land owned by the corporation is worth $5 million. Lulu will own 100% of the corporation and plans to continue operating the ranch for the foreseeable future. Which of the following postmortem planning techniques may Lawrence’s estate elect?

I. Section 303 stock redemption
II. Section 6166 installment payment of federal estate taxes
III. Section 2032A special use valuation
IV. Alternative valuation date

A. Lawrence’s estate is eligible for all the elections shown
B. I, II, III
C. III, IV
D. IV
E. Lawrence’s estate is not eligible for any of the elections shown.

A

ANSWER: B. 35% of Lawrence’s $19 MM adjusted gross estate is $6,650,000. Thus, his estate qualified for 303 and 6166 (35% rule) at $15 million. Section 2032A requires that the real and personal property of the business represent 50% of his gross estate rule ($9.5 million) and 25% of his gross estate in respect to the real property ($4.75 million). Lawrence’s estate does meet the requirements for a Section 2032A election. The Alternate Valuation Date (AVD) is not appropriate because there is no indication of a decline in value of the ranching corporation.

156
Q

CONCEPT: Life Insurance
QUESTION: In 1990 Adam Quincy purchased universal life insurance policy with a single premium of $100,000. In 2005, when the policy’s cash value was $170,000. Adam took a $50,000 loan against the policy. While he paid the annual interest each year, Adam never repaid the loan. This year he died at age 60. If the face value of the universal life (option A level) policy is $1,000,000 and the cash value is $200,000, what amount will his beneficiary be paid?

A. $900,000
B. $950,000
C. $1,000,000
D. $1,050,000
E. $1,150,000 less a 10% penalty on $50,000

A

ANSWER: B. Because this is an option A policy that provides a level death benefit, the amount in the cash value account is immaterial. The insurance company pays only the stated death benefit. The death benefit ($1,000,000) less the existing loan. He paid the penalty when he took out the loan

157
Q

CONCEPT: ISOs
QUESTION: Bob Long is a junior executive with Advanced Knowledge Inc. He is granted ISOs to buy 10,000 shares of his employer’s stock at $10 per share. Bob exercised the options when the FMV of the stock was $20 per share. The exercise caused him to pay $28,000 of AMT. What will be Bob’s basis in the stock when he sells the shares?

A. $0
B. $100,000
C. $128,000
D. $200,000

A

ANSWER: C. Bob had an out-of-pocket cost of $10 per share (the grant price). He also paid $2.80 per share in AMT. The AMT paid increases his basis. Thus, his basis becomes $12.80 per share or $128,000.

158
Q

CONCEPT: Types of Returns (Investment)
QUESTION: Joey and Sally World are both approaching 60. Both Joey and Sally have always worked at low paying jobs. As a result, they are a 10- or 12%-income tax bracket. They have saved sporadically through the years through personal IRAs and non-qualified investment accounts. They never had an opportunity to participate in a company sponsored retirement plan. They have never been to a financial planner for advice. You, a CFP practitioner, are serving them pro bono. They asked you what type of return on their investments would be most meaningful to them. How would you respond?

A. Time weighted return
B. Real rate of return
C. IRR (internal rate of return)
D. Nominal rate of return

A

ANSWER: C. Internal rate of return (IRR) expresses the effective return on an investment considering all cash flows. Time weighted return expresses manager performance. Real return adjusts nominal return for inflation. Nominal return often fails to accurately express a particular investor’s return.

159
Q

CONCEPT: Contribution Limits (Retirement)
QUESTION: Janice is age 51. She is single. She plans to continue to work until age 62. She works for a not-for-profit association which permits employees to participate in 403(b) and 457 plans. Janice wants to contribute to only one plan. Which plan will allow Janice to make the largest tax-deductible contribution?

A. 403(b) plan
B. 457 plan
C. They are both the same.
D. It depends on how much she makes with either employer.
E. She should contribute to both plans up to the maximum.

A

ANSWER: A. Both the 403(b) and the 457 plans offer the same deferral limits. However, because the 457 plan is not provided by a governmental employer no catch-up contribution is available. Janice may make a catch-up contribution to the 403(b) plan. She can only contribute to one plan. Answer D is wrong because there is only one employer.

160
Q

CONCEPT: AGI Calculation (Tax)
QUESTION: Tommy (age 51) and Susan (age 51) Baldwin have been saving for retirement through their 25 years of marriage. They have no children and plan to retire in 4 years. They have provided you with the following information for the current tax year:
Tommy’s W-2 income $200,000
Susan’s W-2 income $175,000
Tommy’s IRA contribution $7,000
Susan’s IRA contribution $7,000
Dividends (qualified & non-qualified) $50,000
Short and long-term gains $75,000
Presuming they file jointly, what is the amount of the Baldwin’s AGI?

A. Without knowing the breakdown of the dividends (qualified vs. non-qualified) and capital gains (short vs. long) there is no solution
B. $486,000
C. $500,000
D. $361,000

A

ANSWER: B. Total W-2 income $375,000
Dividends $50,000
Capital Gains $75,000

Less total IRA
Contributions -14,000
$486,000

There is no indication that either Tommy or Susan is an active participant in a workplace retirement plan. Thus, their AGI does not cause their IRA contributions to phase out for deductibility. Although dividends and capital gains have lower tax rates than most other sources of income, they are part of AGI.

161
Q

CONCEPT: CFP Code of Ethics
QUESTION: The written disclosures under that are required under CFP® Code of Ethics can be made in which of the following ways?

I. Disclosure using the Form ADV
II. Disclosure under the rules and requirement of any self-regulatory organization.
III. Disclosure using multiple written documents
IV. Disclosure used in compliance with state or federal laws.

A. All the above
B. II, IV
C. III, V
D. III

A

ANSWER: A. Under the CFP Code of Ethics, written disclosure to prospective clients may be provided through any of the means shown.

162
Q

CONCEPT: Employee Benefits
QUESTION: Doc Holiday owns a dude ranch that operates as an S corporation. He employs about 5-6 full-time and 6-7 part-time employees. His business includes a bed-and-breakfast operation. He operates as Holiday, Inc. as S corporation. One employee, Jesse, not only breaks in the wild horses but is a wiz at picking the right horse for each dude ranch guest. He cannot afford to lose Jesse as an employee. Other than health insurance Doc provides no other benefits for employees. Doc only takes $2,000 a month as salary and the profit comes out to him as K-1 income. He wants a recommendation from you as to what kind of benefit should he offer to Jesse. What would you suggest?

A. Establish a non-qualified deferred compensation plan for Jesse using a variable annuity.
B. Establish a 162 “double bonus” arrangement using a fixed annuity or life insurance.
C. Establish a SEP presuming that the 3 out of 5 years of service eligibility rule would eliminate most of the employees.
D. Establish a nonqualified deferred compensation arrangement that would operate through a secular trust.

A

ANSWER: B. With the 162 double bonus arrangement, the first bonus would go into the annuity/life insurance and the second would pay Jesse’s phantom tax. Due to its conduit taxation an S corporation cannot offer a non-qualified deferred compensation plan. Since we know little about how long other employees have worked for Doc, it is uncertain as to whether the SEP would eliminate most of the full and part-time employees. The deferred compensation arrangement through a secular trust would create undesirable phantom income for Jesse.

163
Q

CONCEPT: Federal Gift Tax
QUESTION: Jack and Jill Jones are divorcing. In exchange for her release of all marital rights that she has or may have in his estate Jack understands that he will, under the terms of the divorce decree give Jill $1,000,000 as a lump-sum settlement. Under this circumstance, is the $1,000,000 subject to federal gift tax?

A. No
B. No, if the payment occurs within a 3-year period after the divorce
C. Yes
D. Yes, if it is a gift of a future interest
E. Yes, if it is required under the divorce decree

A

ANSWER: B. The $1,000,000 transfer is not subject to gift tax if the transfer of funds occurs within three years of the date of the divorce decree. It is still regarded as a marital gift and thus it is not subject to federal gift tax.

164
Q

CONCEPT: Estate Planning
QUESTION: Jack married Carla in 2016. It is a second marriage for both spouses. Carla has limited assets but is not poor. Jack did not enter into a pre-nuptial agreement with Carla, but they have agreed to basic estate planning goals including:
* Provide income for her through her remaining lifetime
* Leave the principal balances of his assets /accounts to his children
Jack is worth approximately $20 million of which $12 million is in an IRA rollover account. How would you advise Jack to handle his IRA in his estate plan?

A. The IRA should be allocated entirely to a QTIP trust.
B. The IRA should be allocated entirely to a by-pass (B) trust.
C. An amount from the IRA equal to the current estate property exemption amount should be allocated to a by-pass (B) trust the remainder to ae QTIP trust.
D. Leave the IRA outright to Carla naming his children as the contingent beneficiaries.

A

ANSWER: C. It makes sense that up to the exemption equivalent amount is allocated to the B trust. Carla may receive the IRA distributions through the trust. Amounts in the IRA that exceed the exemption equivalent, may then be allocated to a QTIP (C) trust. If $12 million from Jack’s IRA is allocated to a by-pass trust, that trust will be overfunded. Allocating the entire IRA to the QTIP misses the opportunity to maximize the exemption equivalent. If Carla receives the IRA outright, she can transfer it to her own name and change the beneficiary.

165
Q

CONCEPT: Capitalization Value
QUESTION: George Able, age 60, enjoys working and has no wish to retire. He is researching businesses he could buy. Given that entrepreneuring is risky, George’s required rate of return is 7% to 8%. If a business earned an annual net profit of $100,000, what should George pay for the business?

A. $1,000,000
B. $1,250,000
C. $1,333,333
D. $1,428,571

A

ANSWER. B. Choose the exact amount, not less. Using the higher rate 8% produces a lower number ($1,250,000). George would prefer to pay less to acquire the risky business. Using the 7% produces a maximum number. ($1,428,571)

$100,000 = $1,250,000
.08

166
Q

CONCEPT: Gross Income
QUESTION: Which of the following income items are generally not included in gross income?

A. Awards
B. Back pay
C. Bargain purchases of goods from the employer to the extent that the discount exceeds the employers gross profit percentage
D. Death benefits paid by the employer
E. Incentive stock options

A

ANSWER: E. ISOs create AMT income, rather than gross income for Form 1040 purposes. Employer-provided death benefits are not life insurance benefits. The employer will pay and deduct the expense. The other items would be included in a taxpayer’s gross income.

167
Q

CONCEPT: AGI and Deductions
QUESTION: Chris Towns is self-employed. His business employs 10 workers. He is successful enough to fund various benefits for himself and for his employees. To determine AGI, which of the following benefits are either include in gross income or deducted on the front of Form 1040 to determine AGI?
I. The premium for the group life insurance benefit of $220,000 for Chris
II. The annual SEP contribution for Chris
III. Net profit from the sole proprietorship business of $250,000
IV. The IRA contribution by Chris
V. The group health insurance plan premium for Chris

A. All the above
B. I, II, III, V
C. I, II, III
D. II
E. III

A

ANSWER: B. The face value of the group life insurance for Chris is in excess of $50,000. The employer-provided premium that funds the excess is deemed to be compensation. The SEP contribution and the self-employed health insurance premiums are deductions for AGI. The net profit is taxable income. Chris’s IRA contribution is non-deductible because he is an active participant in the SEP and AGI is above the IRA contribution deductibility phaseout.

168
Q

CONCEPT: Financial Planning Process
QUESTION: Mr. and Mrs. Couch have been referred to you. You find out that another financial planner told them that they could retire at age 60. Now at age 55 Mr. Couch realizes that the investment advisor did not get them out of the market in the 2008 decline. Then in 2009 until now the advisor tried make up for the negative year with aggressive decisions that produced both positive and negative returns for specific years. After reviewing the Couch’s data, it is clear to you that retirement at age 60 is no longer a realistic goal. As a CFP® professional, how should you proceed?

A. Tell Mr. Couch to hire an attorney and go to arbitration against the prior advisor.
B. Tell the Couches that you will construct a budget to help them save more money every month.
C. Discuss various financial alternatives that could be acceptable to increase returns and savings to figure out the best course of action.
D. Suggest that Mr. Couch extend his intended retirement date and ask for the client’s thoughts.

A

ANSWER: C. A CFP professional should explore all possible alternatives in order to help the client identify and achieve reasonable retirement objectives. This analysis might include a budget, comparing alternative investment vehicles, extending the retirement date and more. Pursuing arbitration is a long shot in increasing the Couch’s retirement income.

169
Q

CONCEPT: Charitable Gifting/Sale
QUESTION: Terry owns a vacant lot next to the church he attends. He permits worshipers to park on the lot while they attend church services. The church wants to expand and wants to purchase Terry’s land. A certified appraiser reported that the land has an FMV of $1,000,000. Terry is willing to sell the land for $750,000 to the church and claim a $250,000 charitable income-tax deduction. If the basis of the land is $200,000, how much gain will Terry be required to report on the sale of the land?

A. $50,000
B. $250,000
C. $500,000
D. $550,000
E. $600,000

A

ANSWER: E.
$750,000 proceeds x $200,000 basis = $150,000
$1,000,000 FMV

The $750,000 in sales proceeds less $150,000 adjusted basis equals $600,000
However, under a bargain sale to charity the basis is adjusted because the difference between the FMV of the property ($250,000) represents a charitable gift rather than a sale.

170
Q

CONCEPT: FSA
QUESTION: Which of the following statements is true about a medical expense flexible spending account (FSA)?

A. If there is money left in the care account at the end of the year, it can be used for dependent care.
B. An FSA can discriminate in favor of HCEs if less than 70% of the employees participate in the plan.
C. The employee must elect in writing the amount of the salary reduction in the calendar year prior to the year in which that reduction is to take effect.
D. Funds that are forfeited when an employee fails to withdraw them during the allotted time revert directly to the employer for general operations.

A

ANSWER: C. A participant in an FSA plan must elect in writing the amount of the salary reduction in the calendar year prior to the year in which it that reduction is to take effect. Unused funds in medical expense FSAs cannot be used for dependent care. An FSA must not discriminate in favor of HCEs. However, there is no 70% participation testing. Forfeited funds in an FSA may not go directly to the employee for general operations but they can be used to defray future administrative expenses of the FSA itself.

171
Q

CONCEPT: Social Security Benefits
QUESTION: Helen is age 57. Over the years she has fulfilled her own 40-credit/quarter requirements to achieve fully insured status for Social Security benefits. Her husband, Sam, age 59, has also achieved fully insured status. He is currently collecting Social Security disability benefits. Helen had to retire. Her son is unable to work due to physical problems. He is deemed disabled by Social Security since childhood. Which of the following is true?

A. Helen would not be eligible for Social Security benefits because her husband is only 59 and currently receiving Social Security benefits.
B. 85% of her son’s Social Security benefits will be subject to federal income tax.
C. Helen is eligible to receive Social Security benefits because her son is disabled.
D. Helen cannot receive Social Security benefits until she is 62.

A

ANSWER: C. Helen has a child “in care”. “In care” means that the mother performs personal services for child under age 16 or a disabled child. She is also the spouse of a disabled uninsured worker. Helen and her son are entitled to benefits. The information presented does not indicate whether her benefits will be tax-free or subject to federal income tax.

172
Q

CONCEPT: Calls
QUESTION: Mr. Smart is shorting calls (at-the-money calls). What is his greatest risk?

A. Time running out
B. Market interest rates increasing
C. The upside risk is unlimited.
D. A flat market

A

ANSWER: C. Naked call writing is also called shorting calls. The maximum profit is the premium income. Since the upside potential of a stock’s price is unlimited, the potential loss to the writer is unlimited. A shorter defined is someone who believes that the price of the underlying stock will drop or at worse will not rise (a flat market). Mr. Smart will be able to keep the premium income (reference Live Review Investments page 21).